You are on page 1of 95

BIOSTATISTICS AND

METHODS OF
EPIDEMIOLOGY
Questions
DIRECTIONS: Each item below contains a question or incomplete
statement followed by suggested responses. Select the one best response to
each question.

1. Assuming that mammography 2. The association between low


has a sensitivity of 90% and a birth weight and maternal smok-
specificity of 98% and that consec- ing during pregnancy can be stud-
utive tests are independent, what is ied by obtaining smoking histories
the probability that a woman with from women at the time of the
breast cancer will have a negative first prenatal visit and then subse-
yearly screening mammogram for quently assessing and assigning
two consecutive years? birth weight at delivery according
a. 1/10 to smoking histories. What type of
b. 2/10 study is this?
c. 4/10 a. Clinical trial
d. 1/100 b. Cross-sectional
e. 4/100 c. Prospective cohort
d. Case-control
e. Retrospective cohort

Terms of Use
2 Preventive Medicine and Public Health

3. An investigator wishes to per- 5. Among women aged 18 to 34 in


form a randomized clinical trial to a community, weight is normally
evaluate a new beta blocker as a distributed with a mean of 52 kg
treatment for hypertension. To be and a standard deviation of 7.5 kg.
eligible for the study, subjects What percentage of women will
must have a resting diastolic blood have a weight over 59.5 kg?
pressure of at least 90 mm Hg. a. 2%
One hundred patients seen at the b. 5%
screening clinic with this level of c. 10%
hypertension are recruited for the d. 16%
study and make appointments e. 32%
with the study nurse. When the
nurse obtains their blood pressure 6. In nine families surveyed, the
two weeks later, only 65 of them numbers of children per family
have diastolic blood pressures of were 4, 6, 2, 2, 4, 3, 2, 1, and 7.
90 mm Hg or more. The most The mean, median, and mode
likely explanation for this is numbers of children per family are,
a. Spontaneous resolution respectively,
b. Regression toward the mean a. 3.4, 2, 3
c. Baseline drift b. 3, 3.4, 2
d. Measurement error c. 3, 3, 2
e. Hawthorne effect d. 2, 3.5, 3
e. 3.4, 3, 2
4. Which of the following mea-
sures is used frequently as a denom-
inator to calculate the incidence rate
of a disease?
a. Number of cases observed
b. Number of new cases observed
c. Number of asymptomatic cases
d. Person-years of observation
e. Persons lost to follow-up
Biostatistics and Methods of Epidemiology 3

7. A study is undertaken to determine whether drinking more than eight


cups of coffee a day is associated with hypertension. The blood pressure
readings were taken of persons who drink more than eight cups and per-
sons who drink no coffee. The results are as follows:

Hypertension Normal Blood Pressure Total

>8 cups 6 4 10
No coffee 2 7 9
8 11 19

Which of the following is the most appropriate test to analyze the data?
a. Chi-square test
b. McNemars test
c. Fishers exact test
d. Student t test
e. Analysis of variance

Items 810
The results of a study of the incidence of pulmonary tuberculosis in a
village in India are given in the following table. All persons in the village are
examined during two surveys made two years apart, and the number of
new cases was used to determine the incidence rate.

Category of Household Number of Number of


at First Survey Persons New Cases

With culture-positive case 500 10


Without culture-positive case 10,000 10

8. What is the incidence of new cases per 1000 person-years in house-


holds that had a culture-positive case during the first survey?
a. 0.02
b. 0.01
c. 1.0
d. 10
e. 20
4 Preventive Medicine and Public Health

9. What is the incidence of new Items 1213


cases per 1000 person-years in
About 1% of boys are born
households that did not have a
with undescended testes. To deter-
culture-positive case during the
mine whether prenatal exposure to
first survey?
tobacco smoke is a cause of un-
a. 0.001 descended testes in newborns, the
b. 0.1
mothers of 100 newborns with
c. 0.5
undescended testes and those of
d. 1.0
e. 5.0 100 newborns whose testes had
descended were questioned about
10. What is the relative risk of smoking habits during pregnancy.
acquiring tuberculosis in house- The study revealed an odds ratio of
holds with a culture-positive case 2.6 associated with exposure to
compared with households with- smoke, with 95% confidence inter-
out tuberculosis? vals (CI) from 1.1 to 5.3.
a. 0.05
12. Some reviewers are concerned
b. 0.5
c. 2.0 that the study may overestimate
d. 10 the association between maternal
e. 20 smoking and undescended testes in
the offspring because of potential
11. In the study of the cause of a a. Confounding
disease, the essential difference be- b. Nondifferential misclassification
tween an experimental study and c. Differential misclassification
an observational study is that in the d. Selection bias
experimental investigation e. Loss to follow-up

a. The study is prospective


b. The study is retrospective
c. The study and control groups are of
equal size
d. The study and control groups are
selected on the basis of history of
exposure to the suspected causal
factor
e. The investigators determine who is
and who is not exposed to the sus-
pected causal factor
Biostatistics and Methods of Epidemiology 5

13. What is the most appropriate 15. As an epidemiologist, you are


conclusion to be drawn from the asked to recommend the type of
study? study appropriate to the needs of
a. There is no association between researchers who would like to
maternal smoking and undescended study the causes of a rare form of
testes in the offspring sarcoma. They have discovered a
b. The study results, if accurate, sug- registry of this form of cancer and
gest that an offspring whose mother have access to the largest database
smoked is about 2.6 times more of patients with this form of cancer,
likely to be born with undescended which, unfortunately, is only a few
testes than an offspring whose
years old. They have funding for
mother did not smoke
c. The p value > 0.05
only one year from the National
d. The 90% confidence interval for Institutes of Health and note the
these results would probably include budget will be tight. What type of
1.0 study design do you recommend?
e. A larger sample size would increase a. Prospective cohort
the confidence interval b. Retrospective cohort
c. Cross-sectional
14. The probability of being born d. Experimental
with condition A is 0.10 and the e. Case-control
probability of being born with con-
dition B is 0.50. If conditions A and 16. If rapidly progressive cancers
B are independent, what is the are missed by a screening test, which
probability of being born with type of bias will occur?
either condition A or condition B a. Lead-time bias
(or both)? b. Length bias
a. 0.05 c. Selection bias
b. 0.40 d. Surveillance bias
c. 0.50 e. Information bias
d. 0.55
e. 0.60
6 Preventive Medicine and Public Health

Items 17-19
Lou Stewells, a pioneer in the study of diarrheal disease, has developed
a new diagnostic test for cholera. When his agent is added to the stool, the
organisms develop a characteristic ring around them. (He calls it the Ring-
Around-the-Cholera [RAC] test.) He performs the test on 100 patients
known to have cholera and 100 patients known not to have cholera with
the following results:

Cholera No Cholera

RAC test + 91 12
RAC test − 9 88
Totals 100 100

17. Which of the following statements is INCORRECT about the RAC test?
a. The sensitivity of the test was about 91%
b. The specificity of the test was about 12%
c. The false negative rate was about 9%
d. The predictive value of a positive result cannot be determined from the preced-
ing information
e. The predictive value of a negative result cannot be determined from the pre-
ceding information

18. Dr. Stewells next performs the test on 1000 patients with profuse diar-
rhea:

Cholera No Cholera

RAC test + 312 79


RAC test − 31 578
Totals 343 657

Which of the following statements is correct?


a. The predictive value of a positive result is 31/343
b. The predictive value of a positive result is 79/312
c. The predictive value of a negative result is 578/(578 + 31)
d. The predictive value of a negative test is 578/657
e. The incidence rate of cholera in this population is 343/1000
Biostatistics and Methods of Epidemiology 7

19. The RAC test achieves wide- 21. In a study of the cause of lung
spread acceptance. However, with cancer, patients who had the dis-
improvements in hygiene, the prev- ease were matched with controls by
alence of cholera gradually falls from age, sex, place of residence, and
35 to 5% of hospitalized diarrhea social class. The frequency of ciga-
patients. Which statement about the rette smoking was then compared
effect of this fall in prevalence is in the two groups. What type of
true? study was this?
a. The change in prevalence will re- a. Prospective cohort
duce the predictive value of a nega- b. Retrospective cohort
tive result c. Clinical trial
b. The predictive value of a positive d. Case-control
result will decline e. Correlation
c. The specificity of the test is likely to
decline Items 22-24
d. The specificity of the test will in-
crease at the expense of its sensi- The incidence rate of lung can-
tivity cer is 120/100,000 person-years for
e. It will have no impact on the pre- smokers and 10/100,000 person-
dictive values of the test years for nonsmokers. The preva-
lence of smoking is 20% in the
20. A randomized clinical trial is community.
undertaken to examine the effect of
a new combination of antiretroviral 22. What is the relative risk of
drugs on HIV viral load compared developing lung cancer for smokers
to usual therapy. Randomization is compared with nonsmokers?
used for allocation of subjects to a. 5
either treatment or control (usual b. 12
care) groups in experimental stud- c. 50
ies. Randomization ensures that d. 100
a. Assignment occurs by chance e. 120
b. Treatment and control (usual care)
groups are alike in all respects 23. What percentage of lung can-
except treatment cer can be attributed to smoking?
c. Bias in observations is eliminated a. 52%
d. Placebo effects are eliminated b. 78%
e. An equal number of persons will be c. 80%
followed in the treatment and con- d. 92%
trol group e. 99%
8 Preventive Medicine and Public Health

24. If the prevalence of smoking in 26. The use of matching as a tech-


the community was decreased to nique to control for confounding is
10%, the excess incidence rate of most appropriate for which type of
lung cancer that could be averted study?
in that community would be a. A large-scale cohort study
a. 11/100,000 b. A case-control study with a small
b. 22/100,000 number of cases
c. 50/100,000 c. A clinical trial with a factorial
d. 60/100,000 design
e. 110/100,000 d. A cross-sectional study with multi-
ple variables
25. The Coronary Drug Project was e. A correlation study with a small
a randomized trial to evaluate the number of countries
efficacy of several lipid-lowering
drugs. The five-year mortality of the Items 27-28
men who adhered to the regimen of An investigator is designing a
clofibrate (i.e., took 80% of their randomized, double-blind, placebo-
medicine or more) was 15%; among controlled clinical trial to see whether
those assigned to the clofibrate vitamin E will prevent lung cancer.
group who were less compliant, it
was 24.6%. The result was highly 27. Which technique is likely to
statistically significant (p < 0.0001). maximize compliance with the allo-
From this one can conclude cated regimen?
a. Clofibrate was very beneficial to the a. Using the placebo
patients who took it reliably b. Performing a run-in phase
b. Clofibrate is not effective unless c. Using intent-to-treat analysis
patients take at least 80% of the d. Double blinding the study
recommended doses e. Limiting the number of subjects
c. Either clofibrate or something asso- enrolled
ciated with taking it reliably is
strongly associated with reduced
total mortality
d. There was a problem with blinding
in this study
e. Only those who were compliant
should be included in the data
Biostatistics and Methods of Epidemiology 9

28. Which is most likely to affect Items 30-32


the validity (source of bias) of the
A research team wishes to in-
study?
vestigate a possible association be-
a. Loss to follow-up tween smokeless tobacco and oral
b. Incidence of lung cancer
lesions among professional baseball
c. Prevalence of smoking in the source
players. At spring training camp,
population
d. α error they ask each baseball player about
e. β error current and past use of smokeless
tobacco, cigarettes, and alcohol,
29. The crude death rate in the and a dentist notes the type and
United States is 150/100,000. The extent of the lesions in the mouth.
crude death rate in a smaller, devel-
oping country is 75/100,000. Based 30. What type of study is this?
on these data, which one of the fol- a. Case-control
lowing statements best explains the b. Cross-sectional
data? c. Prospective cohort
d. Clinical trial
a. The health care system of the devel-
e. Retrospective cohort
oping country is far better than that
in the United States
b. More people die in the United
States because it has a larger popu-
lation
c. Infant mortality in the first week is
higher in developing countries, but
it is not included in the crude death
rate
d. Death rates in the developing coun-
try are lower due to the emigration
effect
e. Crude death rates are usually higher
in developed countries because of a
higher proportion of older persons
in the population
10 Preventive Medicine and Public Health

31. After the players have been questioned about use of smokeless tobacco
and examined for lesions of the mouth, the data on the 146 players are tab-
ulated as follows:

Mouth No
Lesion Lesion Total

User 80 30 110
Nonuser 2 34 36
Total 82 64 146

In this study, which measure of disease occurrence can be calculated?


a. Incidence rate
b. Cumulative incidence rate
c. Incidence density
d. Prevalence
e. Relative risk

32. Which of the following statements is true?


a. The odds ratio is equal to (80/110) × (2/36) = 13.1
b. A temporal association between smokeless tobacco use and oral lesions can be
established
c. The statistical association can be calculated using the chi-square test
d. Selection bias could overestimate the result
e. There should be an equal number of exposed and nonexposed subjects

33. A randomized trial shows that a new thrombolytic agent reduces total
mortality by 30% in the first 30 days after a suspected myocardial infarc-
tion compared with a placebo (p = 0.002). Which of the following ques-
tions would be the most important to have answered?
a. Was the trial blinded?
b. What was the power of the study?
c. What happened to surviving patients in the next year?
d. What percentage of patients in each group actually had a myocardial infarction?
e. What was the effect on mortality from coronary heart disease?
Biostatistics and Methods of Epidemiology 11

Items 34-36
In a study of the effectiveness of pertussis vaccine in preventing per-
tussis (whooping cough), the following data were collected by studying
siblings of children who had the disease.

Immunization Status Number of Siblings Number of Cases


of Sibling Contact Exposed to Case among Siblings

Complete 4000 400


None 1000 400

34. What was the secondary attack rate of pertussis in fully immunized
household contacts?
a. 0%
b. 10%
c. 25%
d. 40%
e. 75%

35. What was the protective efficacy of whooping cough vaccine?


a. 25%
b. 40%
c. 75%
d. 90%
e. 99%

36. What was the relative risk of contracting whooping cough in the unim-
munized children compared with the fully immunized children?
a. 0.25
b. 0.5
c. 1.0
d. 2.0
e. 4.0
12 Preventive Medicine and Public Health

37. Decision analyses often include a patients utilities in the determina-


tion of the best decision. These utilities measure
a. Whether a patient favors one decision over another
b. Whether a physician favors one decision over another
c. The difference between a patients decision and the physician s decision
d. The relative value a patient places on a particular outcome
e. The relative likelihood of a particular outcome

38. You have just finished conducting a case-control study to measure the
association between alcohol use and lower respiratory tract infections. The
most appropriate method to control for smoking as a confounder is
a. Matching
b. Restriction
c. Randomization
d. Stratification
e. Multivariate modeling

Items 39-41
Data from an investigation of an epidemic of rubella in a remote village
in Brazil are given in the following table:

Number
Not III
but with
Age Number Number Antibody
Group in Ill Rise Number Percent
(years) Population (Symptomatic) (Asymptomatic) Uninfected Infected

09 204 110 74 20 90
1019 129 70 46 13 90
2039 161 88 57 16 90
4059 78 42 28 8 90
60+ 42 2 2 38 10
Totals 614 312 207 95
Biostatistics and Methods of Epidemiology 13

39. Which expression represents the calculation to determine the inci-


dence of illness (symptomatic cases) for all age groups (as a percentage)?
a. 95/519 × 100% = 18.3%
b. 207/614 × 100% = 33.7%
c. 207/519 × 100% = 39.9%
d. 312/614 × 100% = 50.8%
e. 519/614 × 100% = 84.5%

40. Which expression represents the calculation to determine the percent-


age of infection that is asymptomatic (subclinical)?
a. 95/519 × 100% = 18.3%
b. 207/614 × 100% = 33.7%
c. 207/519 × 100% = 39.9%
d. 312/614 × 100% = 50.8%
e. 519/614 × 100% = 84.5%

41. Based on the age-specific infection rates, when did German measles
previously occur in this village in relation to the current epidemic?
a. 0 to 9 years ago
b. 10 to 19 years ago
c. 20 to 39 years ago
d. 40 to 59 years ago
e. 60 or more years ago

Items 4244
A new test has been developed to screen for ovarian cancer. The fol-
lowing figure illustrates the distribution of values for this test among two
populations.

Normal population

Population
with cancer
Frequency

0 10 15 20 25 30 35 40 45 50 55 60 70 !g/dL
14 Preventive Medicine and Public Health

42. If the researcher chooses val- 44. The researcher decides to use
ues under 30 µg/dL as normal lim- values under 20 µg/dL as normal
its for the test, which of the limits, and the test becomes com-
following statements is true? mercially available. One of your pa-
a. The test will be 100% specific tients has a test result of 27 µg/dL.
b. The test will be 100% sensitive You conclude that
c. Some persons without cancer will a. The patient has cancer of the ovary
test positive b. The patient does not have cancer of
d. There will be some false-positive the ovary
tests c. This is a false-negative test
e. All persons with cancer will have a d. A confirmation test will be needed
positive test as she may or may not have cancer
e. This test is not sensitive enough to
43. If the researcher chooses val- detect cancer
ues under 25 µg/dL as normal lim-
its for the test, which of the 45. You are preparing a report to
following statements is true? present to the Public Health Coun-
a. The test will be 100% specific cil on the declining rates of gonor-
b. The test will be 100% sensitive rhea in your state in both men and
c. No false-negative tests will occur women over the last 10 years.
d. There will be some false-positive Which type of graph would best
tests illustrate the data?
e. All persons with cancer will have a
a. Bar chart
positive test
b. Histogram
c. Pie chart
d. Frequency polygon
e. Line graph
Biostatistics and Methods of Epidemiology 15

46. Consider the following two distribution curves.

Which numerical summary measure would allow you to discriminate


between the two distributions?
a. Median
b. Mean
c. Mode
d. Standard deviation
e. Sample size

47. Consider the following distribution curve.

Which statement best applies to this curve?


a. The mean is a more robust measure of central tendency
b. The median is larger than the mean
c. The data is skewed to the right
d. This is a normal distribution
e. This is a bimodal distribution
16 Preventive Medicine and Public Health

Items 48-50
Five prospective cohort studies were undertaken to examine the asso-
ciation between bacterial vaginosis and delivery of a premature child. The
results of these five hypothetical studies are illustrated in the following fig-
ure and are expressed as relative risks with 95% confidence intervals.

D
E

RR
0.2 0.4 0.6 0.8 1.0 1.2 1.4 1.6 1.8

48. Which study appears to have the smallest sample size?


a. A
b. B
c. C
d. D
e. E

49. Which study has a p value > 0.05?


a. A
b. B
c. C
d. D
e. E

50. Which study appears to be the most precise?


a. A
b. B
c. C
d. D
e. E
Biostatistics and Methods of Epidemiology 17

Items 51-53
Five new herpes simplex virus type 2specific ser ologies are developed
by different research laboratories. The test performance characteristics are
used to create the receiver operator curve (ROC) illustrated in the follow-
ing figure.

y C
1.0

E
Sensitivity

A
x
1.0

51. The x axis represents


a. True negatives
b. Prevalence of disease
c. False-negatives
d. False-positives
e. Positive predictive values

52. The main purpose of the ROC curves in the preceding example is to
a. Determine cut-off points for a new test
b. Compare the diagnostic accuracy of the new tests
c. Assess the utility of a new test in a low-prevalence population
d. Determine the test performance characteristics
e. Determine the cost-effectiveness of a new test
18 Preventive Medicine and Public Health

53. Which of the five tests would be best to use as a diagnostic tool?
a. A
b. B
c. C
d. D
e. E

54. A decision analysis is undertaken in an attempt to determine which


approach, radiation therapy or surgery, is best for the management of
prostate cancer. A sensitivity analysis is plotted on the graph shown in the
following figure. The x axis represents the probability of death from surgery,
and the y axis represents the life expectancy (expected utility) expr essed in
quality-adjusted life years (QALYs).

4.509957
S
S
S
S
Expected utility

S Radiation
R R R R R S
R R R R R R R R R R
S
S
S
S Surgery
S
S
S
S
3.797859 S
.05 0.1 .2
pDIE

Based on this information, you conclude that


a. Radiation therapy is always the best approach
b. Surgery is always the best approach
c. Radiation therapy is the best approach when mortality from surgery exceeds
11%
d. Mortality from surgery does not affect the choice of approach
e. Surgery is the preferred approach when mortality from the procedure exceeds
20%
Biostatistics and Methods of Epidemiology 19

55. A prospective cohort study examining the association between passive


smoking and cervical cancer reveals an odd ratio of 1.3 (95% confidence
interval 0.85.6). The most appr opriate conclusion is that
a. There is a significant association between passive smoking and cervical cancer
b. The null hypothesis is rejected
c. There is a type 1 error
d. The α was set at 0.10
e. A 90% confidence interval would result in a narrower confidence interval

56. Consider the following two-way scatter plot examining the relation-
ship between glomerular filtration rate (GFR) on the y axis and the recip-
rocal of plasma creatinine (1/Cr) on the x axis.

(Adapted, with permission, from Ingelfinger JA, Mosteller F,Thibodeau


LA,Ware JH. Biostatistics in Clinical Medicine, 3rd ed. New York, McGraw-
Hill, 1994: 205.)

This is an example of
a. A correlation analysis with a coefficient between 0 and 1
b. A logistic regression analysis
c. A simple linear regression analysis
d. A multiple regression analysis
e. A correlation analysis with a coefficient between −1 and 0
20 Preventive Medicine and Public Health

57. Consider the results of two hypothetical intervention studies:

Study A Study B

Incidence of mortality in the control group 1.3% 20%


Incidence of mortality in the intervention group 0.6% 9.2%

What is the most useful measure of association in assessing the clinical rel-
evance of these two studies?
a. The relative risk (RR)
b. The relative risk reduction (RRR)
c. The odd ratio (OR)
d. The attributable risk reduction (ARR)
e. The numbers needed to treat (NNT)

58. A hypothetical study examining the association between serum cho-


lesterol (>280) and cardiovascular disease (CVD) demonstrates a crude rel-
ative risk of 3.0. When the data is stratified by gender, the relative risk for
men is 4.0 and the relative risk for women is 1.0. The adjusted risk is 3.0.
The most appropriate interpretation of the results of this study is that
a. Gender is both a confounder and an effect modifier
b. Gender is a confounder only
c. Gender is an effect modifier only
d. Gender is neither a confounder nor an effect modifier
e. Gender is a causal pathway

59. A clinical training program wishes to evaluate the reliability of self-


assessment of clinical skills as a tool for measuring improvement. After a
teaching session, students are asked to rank themselves (on a scale of 1 to 5)
on 10 examination procedures. The preceptor also ranks the students ac-
cording to the same scale. The results of the two assessments are then com-
pared. The most appropriate test statistic to compare results is
a. A Kappa statistics test
b. A student t test
c. A Wilcoxon rank sum test
d. A chi-square test
e. A correlation analysis
Biostatistics and Methods of Epidemiology 21

60. Which of the following tests 62. Consider the following study
can be used to study ordinal data assessing the proportion of patients
from two independent samples presenting with urethritis who were
from a population that is not nor- tested for Chlamydiatrachomatis(CT)
mally distributed? at two different health centers:
a. The student t test
b. The Wilcoxon rank sum test
Health CT Test
c. The chi-square test
Center Yes No
d. The one-way analysis of variance
e. The Mantel-Haenszel method A 220 100
B 150 80
61. Point prevalence studies tend
to overestimate the occurrence of
which of the following diseases? The data is analyzed using the chi-
a. Diseases with a high incidence square distribution to determine if
b. Diseases with a long duration there is a significant difference in
c. Diseases with a high mortality proportions between the two health
d. Diseases with a short duration centers. How many degrees of free-
e. Diseases with a low incidence dom should be used for this distri-
bution?
a. 1
b. 2
c. 3
d. 4
e. 6
22 Preventive Medicine and Public Health

63. Consider the following survival curve for women diagnosed with dis-
ease XYZ.

1.0
0.9
0.8
Proportion surviving

0.7
0.6
0.5
0.4
0.3
0.2
0.1

0 1 2 3 4 5
Years since diagnosis

This curve suggests that the five-year survival rate is


a. 10%
b. 20%
c. 30%
d. 40%
e. 50%
Biostatistics and Methods of Epidemiology 23

Items 64-67 Items 68-69


For each of the following For each of the following ques-
descriptions, match the appropri- tions, choose the appropriate epi-
ate measure of frequency. demiologic term it refers to.
a. Fetal mortality a. Internal validity
b. Infant mortality b. External validity
c. Perinatal mortality c. Precision
d. Neonatal mortality d. Power
e. Postneonatal mortality e. Statistical significance
f. Maternal mortality
68. A study demonstrates that the
64. Number of deaths in the first risk of cardiovascular disease among
neonatal 28 days of life per 1000 live births physicians can be reduced by
mortality in 1 year. (SELECT 1 RATE) aspirin intake. Can the results of this
study be applied to the population
65. Number of fetal deaths plus at large? External validity
perinatal deaths in the first week of life per
mortality 1000 total births in 1 year. (SE- 69. An intervention study dem-
LECT 1 RATE) onstrates that attending a sexual
historytaking skills-building work-
66. Number of deaths under the shop increases the level of comfort
infant age of 1 year per 1000 live births in of providers in questioning pa-
mortality 1 year. (SELECT 1 RATE) tients about the number of sexual
partners (RR = 1.4, 95% CI 1.2
67. Number of deaths between the 33.8). Are the results of the study
postneonatal ages of 28 days and 11 months per reliable? precision
mortality 1000 live births in 1 year. (SE-
LECT 1 RATE)
24 Preventive Medicine and Public Health

DIRECTIONS: Each group of questions below consists of lettered


options followed by numbered items. For each numbered item, select the
appropriate lettered option(s). Each lettered option may be used once,
more than once, or not at all. Choose exactly the number of options indi-
cated following each item.

Items 70-73 73. Comparison of serum choles-


terol before and after ingestion of
For each of the studies below,
hamburgers in a sample of fast-food
choose the most appropriate statis-
patrons. (SELECT 1 TEST)
tical test to analyze the data.
paired t-test
a. Chi-square analysis Items 74-77
b. Student t test
c. Paired t test For each of the following de-
d. Analysis of variance scriptions, pick the appropriate epi-
e. Linear regression demiologic term.
f. Multiple regression
a. Confounding
g. Correlation analysis
b. Effect modification
h. McNemar test
c. Differential misclassification
d. Lead-time bias
70. Comparison of systolic blood e. Selection bias
student pressures in independent samples of f. Nondifferential misclassification
t-test pregnant and nonpregnant women.
(SELECT 1 TEST) 74. Elevated bilirubin levels in neo-
nates are associated with brain dam-
71. Comparison of the prevalence age only in babies who also have
chi-square of hepatitis B surface antigen infections or severe hemolytic dis-
(HBsAg) in medical and dental stu- ease. (SELECT 1 TERM) effect modification
dents. (SELECT 1 TEST)
75. People who drink coffee tend
72. Comparison of the level of to smoke more, and for this reason
ANOVA blood glucose in male and female coffee drinkers have a higher risk of
rats following administration of lung cancer. (SELECT 1 TERM) Confounding
three different drugs. (SELECT 1
TEST) 76. Higher lead levels in hyperac-
tive children may be due to in-
creased consumption of paint in
children who were already hyper-
active. (SELECT 1 TERM)
Selection bias
Biostatistics and Methods of Epidemiology 25

77. A prospective cohort study 81. At the end of the study, a total of
with an imprecise measurement of 129 nonfatal myocardial infarctions
exposure to radiation fails to dem- per 54,560 person-years of observa-
onstrate a significant association tion occurred in the study popula-
with cancer. (SELECT 1 TERM) tion. (SELECT 1 PARAMETER)
Nondifferential Incidence density
Items 78-81 misclassificatio Items 82-86
n data
In each statement below, Match the examples below with
are presented based on a cohort the appropriate epidemiologic terms.
study of coronary heart disease. a. Lead-time bias
Choose the parameter that best de- b. Surveillance bias
scribes each of these statements. c. Recall bias
a. Point prevalence d. Type 1 error
b. Cumulative incidence e. Power
c. Standardized morbidity ratio f. Length time bias
d. Relative risk g. Confounding
e. Incidence density
f. Odds ratio 82. Medical students who fail a
g. Case fatality rate physiology examination are more
likely to report missing two or more
78. At the initial examination, 17 physiology lectures than those who
point persons per 1000 had evidence of fail a neuroanatomy examination.
prevalence coronary heart disease (CHD). (SE- (SELECT 1 TERM) Recall bias
LECT 1 PARAMETER)
83. The chance of discovering the
79. Among a cohort of heavy smok- truth that twice as many of your
ers, the observed frequency of angina friends are at the movies as are
standardized pectoris was 1.6 times as great as the studying for their board examina-
morbidity ratio expected frequency during the first tions. (SELECT 1 TERM) Power
12 years of the study. (SELECT 1
PARAMETER) 84. In a class of 150 medical stu-
dents, there will likely be a few who
80. During the first eight years of can answer this question correctly
the study, 45 persons developed without understanding the mate-
coronary heart disease per 1000 per- rial. (SELECT 1 TERM)
Type 1 error
sons who entered the study free of
disease. (SELECT 1 PARAMETER)
Cumulative incidence
26 Preventive Medicine and Public Health

85. The likelihood of finding a lost 90. Children between the ages of 1
Surveillance biochemistry notebook in your and 5 have an average of eight
bias apartment is higher in the month of colds per year. (SELECT 1 RATE)
June than in the month of March. Morbidity rate
(SELECT 1 TERM) Items 9194
Choose the term that best fits
86. Medical students enrolled in a
the description.
first-year anatomy class are more
Lead time likely to remain at their same ad- a. Matching
bias b. Stratification
dresses for the next two years than
c. Age adjustment
medical students enrolled in fourth- d. Multivariate statistical analysis
year clerkships. (SELECT 1 e. Survival analysis
TERM)
91. In a cohort study of hyperten-
Items 8790 sive men, the proportions of subjects
Choose the rate that best de- with high and low renin levels who
scribes each statement below. survived for five years are compared
a. Secondary attack rate separately among those aged 40 to
b. Case fatality rate 49, those aged 50 to 59, and those
c. Morbidity rate aged 60 to 69 at entry. (SELECT 1
d. Age-adjusted mortality TERM) stratification
e. Crude mortality
92. A sampling strategy is used to
Case fatality 87. Death occurs in 10% of cases achieve comparability of the groups matching
of meningococcal meningitis. (SE- being studied. (SELECT 1 TERM)
rate LECT 1 RATE)
93. A technique that takes into
Crude 88. Approximately 9 people die account variable length of follow-
each year in the United States for up is used. (SELECT 1 TERM) survival analysis
mortality every 1000 estimated to be alive.
(SELECT 1 RATE) 94. Six different risk ratios are cal-
culated: one for each sex at each of
89. Eighty percent of susceptible three social class levels. (SELECT
Secondary household contacts of a child with 1 TERM) stratification
attack rate chicken pox develop this disease.
(SELECT 1 RATE)
Biostatistics and Methods of Epidemiology 27

Items 95-98 97. Acute anxiety neurosis was


diagnosed among 250 patients and
For each of the studies de-
follow-up data were available on
scribed, select the reason for which
80% of these patients 10 years later.
the conclusion can be misleading
The mortality experience of this
or false.
cohort was no different from that
a. Lack of a control group of the general population. The
b. Lack of proper follow-up
authors concluded that the diag-
c. Lack of adjustment for age
d. Lack of denominators
nosis of acute anxiety neurosis is
e. Lack of adjustment for race not associated with a decrease in
longevity. (SELECT 1 ERROR)
95. Of 250 consecutive, unselected
lack of proper follow-up
women in whom acute cholecystitis 98. Of 143 patients who died of
was diagnosed, 75 were under age bacterial endocarditis and on whom
lack of autopsies were performed, 2% were
50 and 175 were over age 50. The
denominators investigator concluded that older less than 10 years of age. The
women are at greater risk of acute authors concluded that bacterial
cholecystitis than are younger endocarditis is rare in childhood.
women. (SELECT 1 ERROR) (SELECT 1 ERROR)
lack of denominators
96. In a review of 3000 patients
in whom adult-onset diabetes was
diagnosed, 2000 of these patients
were obese at the time of diagnosis.
The investigator concluded that
there is an association between
diabetes and obesity. (SELECT 1
ERROR) lack of a control group
28 Preventive Medicine and Public Health

Items 99-102
Consider the following decision tree assessing radiation therapy versus
surgery for the treatment of prostate cancer: The expected utility, life
expectancy, is expressed in quality-adjusted life years, or QALYs.

Life
expectancy
(QALYS)
Death
0
0.05
?
4.4

Surgery Recurrence
3.15
0.10
Incontinence
0.05
No recurrence
4.8 4.50
Survive 0.90
0.95 Recurrence
3.50
0.10
No incontinence
0.95
No recurrence
5.00
0.90

3.9

Recurrence
3.15
0.40
4.2 Proctitis
0.43
No recurrence
4.4 4.50
Radiation 0.60
Recurrence
3.50
0.40
No proctitis
0.57
No recurrence
5.00
0.60
Biostatistics and Methods of Epidemiology 29

99. Which of the following state- 102. If radiation therapy was never
ments is true concerning the cre- associated with the complication of
ation of a decision tree used for proctitis, the quality-adjusted life
clinical decision making? expectancy would be
a. The first nodes in a tree are chance a. 5.0
nodes b. 3.5
b. Branches from the chance nodes c. (3.5 + 5.0)/2 = 4.25
must be mutually exclusive and col- d. (3.15 × 0.4) + (4.5 × 0.6) = 3.96
lectively exhaustive e. (3.5 × 0.4) + (5.0 × 0.6) = 4.4
c. The terminal nodes represent preva-
lence of disease Items 103-106
d. The expected utilities are calculated
by folding back the tree from left to The following 2 × 2 table rep-
right resents the findings of a five-year
e. The numerical values ofthe expected cohort study in which the inci-
utility are expressed in different units dence of suicide in veterans who
than the expected outcomes served in Vietnam was compared
with that of veterans who served
100. The quality-adjusted life ex- elsewhere. Match the name of the
pectancy for surgery is parameter below with the appro-
a. (4.4 + 4.8) × 0.95 = 8.74 priate formula.
b. (4.4 × 0.95) + (4.8 × 0.05) = 4.42
c. [(4.4 × 0.05) + (4.8 × 0.95)] × No
0.95 = 4.5 Suicide Suicide
d. (4.4 × 0.95) + (4.8 × 0.95) = 8.74
e. (4.4 × 0.90) + (4.8 × 0.90) = 8.28 Served in Vietnam a b
Served elsewhere c d
101. Based on the results of this
decision analysis, which approach a. ad/bc odds ratio
appears preferable? b. (a + b)/(a + b + c + d)
a. Surgery c. (a + c)/(a + b + c + d) overall incidence
b. Radiation d. [a/(a + b)]/[c/(c + d)] relative risk
c. Surgery, only if there is no probabil- e. [a/(a + b)] − [c/(c + d)] excess risk of suicide
ity of death
d. Radiation followed by surgery, if 103. The odds ratio. (SELECT 1
there is a recurrence FORMULA)
e. No preferable approach can be iden-
tified 104. The relative risk. (SELECT 1
FORMULA)
30 Preventive Medicine and Public Health

105. The excess risk of suicide in 110. Individuals are divided into
Vietnam veterans. (SELECT 1 subgroups on the basis of specified
FORMULA) characteristics and then random
samples are selected from each sub-
106. The overall incidence (per group. (SELECT 1 PROCEDURE)
five years) of suicide in the study. Stratified sampling
(SELECT 1 FORMULA) Items 111-115
A new test for chlamydial infec-
Items 107-110
tions of the cervix is introduced.
Match each description of a Half of the women who are tested
sampling procedure with the cor- have a positive test. Compared with
rect term. the gold standard of careful cultures,
a. Systematic sampling 45% of those with a positive test are
b. Paired sampling infected with chlamydia, and 95%
c. Simple random sampling of those with a negative test are free
d. Stratified sampling of the infection. Match the epidemi-
e. Cluster sampling ologic terms below with the correct
percentage.
107. Each individual of the total a. 25%
simple random group has an equal chance of being b. 45%
selected. (SELECT 1 PROCE- c. 63%
DURE) d. 90%
e. 95%
108. Households are selected at
random, and every person in each 111. Sensitivity of the test. (SE-
cluster
household is included in the sam- LECT 1 PERCENTAGE) 90%
ple. (SELECT 1 PROCEDURE)
112. Specificity of the test. (SE-
109. Individuals are initially as- LECT 1 PERCENTAGE) 63%
sembled according to some order in
a group and then individuals are 113. Prevalence of chlamydial in-
selected according to some constant fection in that community. (SE-
determinant; for instance, every LECT 1 PERCENTAGE) 25%
fourth subject is selected. (SELECT
1 PROCEDURE) 114. Predictive value of a positive
systematic sampling test. (SELECT 1 PERCENTAGE) 45%

115. Predictive value of a negative


test. (SELECT 1 PERCENTAGE) 95%
Biostatistics and Methods of Epidemiology 31

Items 116-119 119. In a case-control study of


lung cancer, cases spouses ar e cho-
For each result or conclusion
sen as controls. The odds ratio for
described below, select the choice
smoking is 3.0, which does not
that might best explain it.
quite reach statistical significance
a. Ecologic fallacy (N = 30 per group; p = 0.07).
b. Type 1 error
(SELECT 1 ERROR) selection bias
c. Type 2 error
d. Selection bias
e. Misclassification bias Items 120124
For each variable described be-
116. A randomized blinded trial low, choose the type of measure-
of aspirin to prevent myocardial ment scale.
infarction fails to find a difference a. Dichotomous scale
Type 2 error between aspirin and placebo b. Nominal scale
groups after five years (N = 500 per c. Ordinal scale
group; p = 0.11). (SELECT 1 d. Interval scale
ERROR) e. Ratio scale

117. A study of patterns of contra- 120. Survival of a particular patient


ceptive use finds that counties with for at least five years. (SELECT 1
the highest per capita use of con- SCALE) dichotomous
Ecologic fallacy doms also have the highest preg-
nancy rates (N = 100,000; p < 121. Frequency of somnolence
0.001) and concludes that con- during biochemistry lectures: never,
doms are ineffective as contracep- sometimes, usually, or always. (SE-
tives. (SELECT 1 ERROR) LECT 1 SCALE) ordinal

118. An investigator analyzes data 122. Birth weight. (SELECT 1


from the National Health Interview SCALE) ratio
Survey and finds that there is a pos-
Type 1 error
itive association between consump- 123. Type of medical specialty. (SE-
tion of turkey and degenerative LECT 1 SCALE) nomnial
joint disease in black women 50 to
59 years old (N = 50; p < 0.05). 124. Year of birth. (SELECT 1
(SELECT 1 ERROR) SCALE) interval
32 Preventive Medicine and Public Health

Items 125-127 125. The specificity of the Blues


test was 85%. (SELECT 1 PER-
Dr. Vera Blues, a noted psychi-
CENTAGE)
atric epidemiologist, is interested in
the diagnosis of depression. She
126. The likelihood that someone
develops a new test for its diagno-
with depression would have a posi-
sis, which she calls the Blues test.
tive Blues test was 80%.
According to the gold standard,
(SELECT 1 PERCENTAGE)
which involves meeting the DSM-
IV criteria, about 10% of adults in
127. The likelihood that someone
the United States are depressed. Dr.
in the population with a negative
Blues applies her new test to 100
Blues test would be depressed was
persons diagnosed as being de- < 10%. (SELECT 1 PERCENT-
pressed by the gold standard; 80
AGE)
have a positive Blues test. She finds
400 persons who are not de-
pressed; 60 have a positive test. She
reports her findings in the Journal
of the Society of Academic Psychia-
trists (JSAP). Match the statements
that Dr. Blues made in her article
with the appropriate percentage.
a. 85%
b. 80%
c. 60%
d. 6%
e. <10%
Biostatistics and Methods of Epidemiology 33

Items 128-130
Consider the following portion of a decision tree assessing the screen-
ing strategies with different tests for Chlamydia trachomatis.

Infection
Test positive
No infection

DNA-probe

Infection
Test negative
No infection

For each probability described below, select the most appropriate


definition.
a. True positives
b. Prevalence of disease
c. True positives + false-positives
d. True positives + false-negatives
e. True negatives + false-negatives
f. True negatives + false-positives
g. Positive predictive value
h. Negative predictive value
i. 1− negative predictive value
j. 1− positive predictive value

C 128. The probability of a positive DNA-probe. (SELECT 1 DEFINITION)

I 129. The probability of infection if the DNA-probe is negative. (SELECT


1 DEFINITION)

J 130. The probability of no infection if the DNA-probe is positive.


(SELECT 1 DEFINITION)
EPIDEMIOLOGY AND
PREVENTION OF
COMMUNICABLE DISEASES
Questions
DIRECTIONS: Each item below contains a question or incomplete
statement followed by suggested responses. Select the one best response to
each question.

131. A 6-year-old child is brought 132. Which of the following con-


to the emergency room by her par- ditions has been associated with a
ents on a Friday night because they false-positive Fluorescent Trepone-
are concerned about rabies. A bat mal Antibody Absorption (FTA-
was present in the childs bedroom ABS) test?
when they arrived at their country a. Tuberculosis
home that evening. It started flying b. Mononucleosis
around the head of the girl when she c. Lyme disease
entered her room and it ruffled her d. Viral pneumonia
hair. The parents heard her scream, e. HIV infection
ran up to her room, and shooed the
bat out the window. Upon examina- 133. One of your patients, a 30-
tion, there is no visible bite or scratch year-old developer, tells you he is
marks. Which is the most appropri- planning a trip to the Dominican
ate intervention at this time? Republic the following month. He
a. Reassure the parents that there is no
will need to travel in rural areas.
risk of rabies given the history and Which is the most appropriate in-
examination tervention for malaria prophylaxis
b. Consult public health authorities for this patient?
to determine the epidemiology of a. No prophylaxis
rabies in that area b. Chloroquine
c. Administer rabies vaccine and rabies c. Mefloquine
immunoglobulin (RIG) d. Doxycycline
d. Administer rabies immunoglobulin e. Primaquine
(RIG) only
e. Administer rabies vaccine only
69

Terms of Use
70 Preventive Medicine and Public Health

134. A 20-month-old child pre- 136. Which of the following vac-


sents to your office with a mild viral cines is CONTRAINDICATED dur-
infection. The results of examination ing pregnancy?
are normal except for a temperature a. Hepatitis B vaccine
of 37.2°C (99°F) and clear nasal dis- b. Varicella vaccine
charge. Review of her vaccination c. Influenza vaccine
records reveals that she received only d. Tetanus toxoid
two doses of polio vaccine and e. Rabies vaccine
diphtheria-tetanus-pertussis (DTaP)
vaccine, and that she did not 137. A 32-year-old farmer pre-
receive the measles-mumps-rubella sents to the emergency room with a
(MMR) vaccine. The mother is 20 crushing injury of the index finger
weeks pregnant. Her brother is and thumb that occurred while
undergoing chemotherapy for leu- he was working with machinery in
kemia. Which of the following is his barn. Records show that he re-
the most appropriate intervention? ceived three doses of Td in the past,
a. Schedule a visit in two weeks for and that his last dose was given
DTaP when he was 25 years old. In addi-
b. Administer inactivated polio vac- tion to proper wound cleaning and
cine (IPV) and DTaP management, which of the follow-
c. Administer DTaP, oral polio vaccine ing is the most appropriate preven-
(OPV), and MMR tion intervention?
d. Administer DTaP, IPV, and MMR a. No additional prophylaxis
e. Administer DTaP and OPV and b. Administration of tetanus toxoid
schedule a visit in three months for c. Administration of tetanus immuno-
MMR globulin only
d. Administration of tetanus toxoid
135. Prevention of human brucel- and immunoglobulin
losis depends primarily on e. Administration of tetanus and diph-
a. Pasteurization of dairy products de- theria toxoid
rived from goats, sheep, or cows
b. Treatment of human cases
c. Control of the insect vector
d. Immunization of farmers and
slaughterhouse workers
e. Destruction of infected animals
Epidemiology and Prevention of Communicable Diseases 71

138. Epidemics of typhus fever 141. Professional organizations rec-


have been associated with war and ommend that all pregnant women
famine for several centuries. What be routinely counseled about HIV
factor was most important in the infection and be encouraged to be
control of such epidemics follow- tested. What is the most important
ing the end of World War II? reason for early identification of HIV
a. Eradication of Anopheles mosqui- infection in pregnant women?
toes a. A cesarean section can be planned
b. Improved sanitation practices to reduce HIV transmission to the
c. Improved methods for handling newborn
food supplies b. Breast feeding can be discouraged
d. Disinfestation by use of DDT to reduce transmission to the new-
e. Mass therapy with antibiotics born
c. Early identification of a newborn at
139. Immunization of preschool risk of HIV infection will improve
children with diphtheria toxoid survival
results in d. Counseling on pregnancy options,
such as termination, can be offered
a. Protection against the diphtheria
e. Antiretroviral therapy can be offered
carrier state
to reduce the chance of transmission
b. Lifelong immunity against diphthe-
of HIV to the newborn
ria
c. Detectable antitoxin or immuno-
logic memory for about 10 years 142. A 35-year-old patient comes
d. Frequent adverse reactions to your office in early April for a
e. Protection against infection of the routine examination. In the course
respiratory tract by Corynebacte- of the history, he tells you that he
rium diphtheriae plans to go turkey hunting in Nan-
tucket, Massachusetts, for one week
140. What is the recommended in May. He is concerned about Lyme
interval in months between the ad- disease. Which is the most appro-
ministration of whole blood transfu- priate intervention for preventing
sion and the measles-mumps-rubella Lyme disease?
(MMR) vaccine? a. Vaccination
a. 0 b. Avoidance of bushy areas
b. 1 c. Tick check at the end of each day
c. 3 d. Protective clothing and DEET
d. 6 e. Antibiotic prophylaxis for one week
e. 10
72 Preventive Medicine and Public Health

Items 143-144 145. In the course of investigating


a 24-year-old HIV-infected male,
An 18-year-old sexually active
the HBsAg is positive. He is cur-
college student presents with com-
rently asymptomatic, his physical
plaints of lower abdominal pain
examination is essentially normal,
and irregular bleeding for five days.
and his CD4 cell count is 800.
She has no fever. She uses oral con-
Which of the following tests is most
traceptives as method of birth con-
helpful in determining whether the
trol. Upon examination, the cervix
patient is in the acute phase of viral
is friable, there is cervical motion
hepatitis?
tenderness and adnexal tenderness.
The pregnancy test is negative. a. ALT levels
b. HBeAg
c. HBsAg
143. Which is the most likely etio-
d. IgG anti-HBcAg
logic agent responsible for these e. IgM anti-HBcAg
findings?
a. Neisseria gonorrhoeae Items 146-148
b. Chlamydia trachomatis
c. Treponema pallidum You are a newly employed phy-
d. Herpes simplex virus type 2 sician at a community hospital and
e. Mycoplasma hominis have been given the responsibility
of overseeing the infection control
144. She tells you that she had a program. You plan to conduct a
similar episode two years ago. What prospective surveillance of nosoco-
is her risk of infertility following mial infections of patients, hire in-
this second clinical episode of fection control personnel, and begin
pelvic inflammatory disease? an educational program for hospital
a. <1% personnel.
b. 5%
c. 10% 146. Based on national data, you
d. 20% expect that the incidence of noso-
e. 40% comial infections in your facility
will be
a. <1%
b. 1 to 2%
c. 3 to 5%
d. 6 to 8%
e. 9 to 10%
Epidemiology and Prevention of Communicable Diseases 73

147. You expect the most com- 150. Which patient is most likely
mon site of infection to be to become a chronic carrier follow-
a. Urinary tract ing an acute episode of hepatitis B?
b. Surgical wounds a. A newborn
c. Respiratory tract b. A 20-year-old female following
d. Bloodstream vaginal sexual transmission
e. Gastrointestinal tract c. A 50-year-old male following rectal
sexual transmission with a partner
148. The intervention most likely positive for HBeAg
to decrease the transmission of nos- d. A 30-year-old health care worker
ocomial infections in your institu- following a percutaneous injury
tion is e. A 40-year-old HIV-infected male
with a CD4 cell count of 200
a. Adding proper ventilation systems
b. Disinfecting sheets and towels
c. Decreasing the use of indwelling
Items 151-153
catheters A 2-year-old child is brought
d. Enforcing adherence to hand wash- to the emergency room with severe
ing prostration, a temperature of 40°C
e. Eliminating common sources of in-
(104°F), and a few petechial lesions
fection
around the ankles. She had mild
upper respiratory symptoms until
149. In the United States, the larg-
her condition started deteriorating a
est proportion of tuberculosis cases
few hours before. A Gram stain on
occurs among
the buffy coat of blood reveals gram-
a. HIV-infected persons negative diplococci. Treatment is
b. Injecting drug users
promptly initiated.
c. Homeless persons
d. Foreign-born persons
e. Incarcerated persons 151. The case fatality rate for this
clinical manifestation of disease is
a. Less than 5%
b. 5 to 15%
c. 20 to 30%
d. 40 to 50%
e. Greater than 50%
74 Preventive Medicine and Public Health

152. Compared with the general 155. Which sexual partners should
population, the risk of developing be informed of the exposure and
an infection among household con- referred for evaluation?
tacts is a. Current sexual partners only
a. The same b. Partners of within 30 days
b. 10 to 20 times greater c. Partners of within 60 days
c. 50 to 100 times greater d. Partners of within 90 days
d. 200 to 400 times greater e. Partners of within 120 days
e. 500 to 800 times greater
156. A 7-year-old girl is brought to
153. The child had been attending your office by her mother because of
a day care center. In addition to rec- a rash that appeared three days ago.
ommending close surveillance for Her temperature is 37.2°C (99°F)
early signs of illness, which of the and her face has an intense rash with
following is the most appropriate a slapped-cheek appearance. The
management of day care contacts? most likely etiologic agent is
a. No further action a. Adenovirus
b. Vaccination of children only b. Rotavirus
c. Vaccination of children and adults c. Parvovirus
d. Antibiotic prophylaxis of children d. Coxsackievirus
only e. Echovirus
e. Antibiotic prophylaxis of adults
and children 157. To which patient would the
MMR be safe to administer?
Items 154-155 a. A 15-month-old HIV-infected child
A 25-year-old man presents with a CD4 cell count of 700
with a single, indurated, painless ul- b. A 25-year-old pregnant woman
c. A 12-year-old asthmatic on 20 mg
cer on the penis that appeared two
of oral prednisone daily for the last
days ago. His most recent unpro-
20 days
tected sexual contact was 21 days d. An 18-year-old with leukemia in
before. An immediate rapid plasma remission whose chemotherapy
reagin (RPR) test is negative. was terminated 1 month ago
e. A 17-year-old with a life-threatening
154. The most likely diagnosis is anaphylactic reaction to egg inges-
a. Syphilis tion
b. Herpes
c. Chancroid
d. Lymphogranuloma venereum
e. Donovanosis (granuloma inguinale)
Epidemiology and Prevention of Communicable Diseases 75

Items 158-159 160. A 19-year-old college student


presents to the university student
On a Friday afternoon, a 30-
health center complaining of severe
year-old nurse is brought to
coughing spells for the last four
employee health for evaluation fol-
days, following initial symptoms of
lowing a needle-stick injury that
coryza and malaise. She is afebrile.
occurred at the AIDS clinic. The
Her medical history is uneventful,
source patient is known to be in-
and immunizations are up to date.
fected with HIV and has advanced
She is a member of the basketball
AIDS.
team. During weekends, she baby-
sits a 10-month-old and a 2-year-
158. Which of the following fac-
old. In terms of management of
tors carries the greatest risk for
contacts, which etiological agent is
transmission of HIV to the health
the most important to include in
care worker?
the differential diagnosis?
a. Depth of the injury
a. Streptococcus pneumoniae
b. Stage of illness of the source patient
b. Mycoplasma pneumoniae
c. Presence of visible blood on the
c. Bordetella pertussis
needle
d. Influenza virus
d. Use of gloves during the procedure
e. Legionella pneumophila
e. Entrance of the needle into a vein
or artery of the source patient
161. Which of the following infec-
159. Which is the most appropri- tions is transmitted chiefly from
ate course of action for this health person to person?
care worker? a. California encephalitis
b. St. Louis encephalitis
a. Reassure her of the low risk of in-
c. West Nilelike viral encephalitis
fection and offer no prophylaxis for
d. Meningococcal meningitis
HIV infection
e. Eastern Equine Encephalitis (EEE)
b. Offer single-drug antiretroviral ther-
apy
c. Offer two-drug antiretroviral ther-
apy
d. Offer triple-drug antiviral therapy
e. Draw an HIV antibody test and refer
her to the infectious disease special-
ist first thing Monday morning
76 Preventive Medicine and Public Health

162. Widespread use of the Haemo- 164. As an epidemiological inves-


philus influenzae type b vaccine has tigation officer for the Centers for
resulted in a dramatic decrease in Disease Control and Prevention,
the number of cases of meningitis you are contacted by a local health
due to this bacterium. Which agent department. They inform you that
is now the leading cause of bacterial a large number of persons have
meningitis in children in the United acquired mild symptoms of influ-
States? enza despite being vaccinated for
a. Streptococcus pneumoniae the appropriate strain being cul-
b. Group B Streptococcus pyogenes tured. You find that the cultured
(hemolyticus) strain is the same as that incor-
c. Non-type-b Haemophilus influenzae porated into the trivalent vaccine
d. Escherichia coli K-1 administered throughout the world.
e. Neisseria meningitidis You also note that the strain had a
high case fatality rate in previous
163. The medical evaluation of a epidemics in China, where most
25-year-old intravenous drug user new strains are isolated and identi-
reveals elevated liver enzymes and fied for vaccine preparations. The
a positive anti-HBsAg. The most most likely explanation for the out-
likely cause of the abnormal liver break noted by the local health
profile is hepatitis department is
a. A a. Vaccine failure
b. B b. Antigenic drift
c. C c. Antigenic shift
d. D d. Herd immunity
e. E e. Incomplete immunity from previ-
ous rhinovirus infections
Epidemiology and Prevention of Communicable Diseases 77

165. A 38-year-old HIV-infected woman presents for follow-up evalua-


tion. She is on antiretroviral therapy. She has no complaints. Her physical
examination is normal. Her PPD is reactive at 2 mm. The chest x-ray is nor-
mal. She has no history of past TB or recent known contact with infectious
TB. She lives at home alone. Her CD4 + T cell count is 180/µL. Her previ-
ous count was 175/µL. Prophylaxis is most appropriate for which of the
following infections?
a. Mycobacterium avium complex (MAC)
b. Cryptococcus neoformans
c. Mycobacterium tuberculosis
d. Toxoplasma gondii
e. Pneumocystis carinii

166. During the investigation of an outbreak of food poisoning at a sum-


mer camp, food histories were obtained from all campers as indicated in
the following table. Which of the food items was probably responsible for
the outbreak?

Proportion of Campers Who Developed


Illness (Percent)
Campers Who Ate Campers Who Did Not Eat

Food Specified Food Specified Food


a. Hamburger 61 48
b. Potatoes 70 35
c. Ice cream 40 50
d. Chicken 73 10
e. Lemonade 20 45

167. In 1999, the majority of cumulative cases of AIDS in the United


States occurred in which exposure category?
a. Men who have sex with men
b. Users of intravenous drugs
c. Women who have sex with women
d. Hemophiliacs
e. Persons who engage in heterosexual contact
78 Preventive Medicine and Public Health

168. Which of the following com- Items 171-173


plications has been associated with
As medical director of a divi-
the recall of rotavirus vaccine?
sion of epidemiology in a state
a. Guillain-Barr syndr ome health department, you are asked to
b. Hemolytic anemia
develop a hepatitis C awareness
c. Febrile seizures
campaign. You develop a document
d. Intussusception
e. Neutropenia with answers to the most frequently
asked questions (FAQ) by medical
169. The time interval between providers. You follow the 1999 CDC
entry of an infectious agent into a recommendations.
host and the onset of symptoms is
called 171. Which group should you rec-
ommend for routine screening?
a. The communicable period
b. The incubation period a. Pregnant women
c. The preinfectious period b. Emergency medical personnel
d. The noncontagious period c. Health care workers
e. The decubation period d. Persons who ever injected illegal
drugs
e. Household contacts of HCV-positive
170. An 8-year-old child is brought
persons
to the emergency room with pro-
fuse, bloody diarrhea. The symp-
172. Which test should you rec-
toms started about three days ago,
ommend for screening?
but gradually worsened. He has no
fever. His platelet count is 40,000. a. EIA for anti-HCV
b. Immunoblot assays
The most likely source of the enteric
c. Qualitative HCV RNA
infection is d. Quantitative HCV RNA
a. Fish e. ALT levels
b. Chicken
c. Milk
d. Eggs
e. Beef
Epidemiology and Prevention of Communicable Diseases 79

173. What is the most appropriate 175. The most likely source of
counseling message to offer to HCV- infection is
positive pregnant women? a. A coworker
a. Cesarian section should be per- b. Food
formed c. His wife
b. The probability of transmission to d. Water
the newborn is 5% e. His children
c. Breast feeding should be discour-
aged 176. A 10-year-old boy with sickle
d. Infants should receive IgG at birth cell disease presents with headache,
e. Infants often do poorly in the first anorexia, and fever. He complains
years of life of pain in the right tibia and local
inflammation is noted. Osteo-
Items 174-175 myelitis is diagnosed. The most
You are a public health physi- likely etiologic agent is
cian working at a city health de- a. Listeria
partment and receive a report of a b. Salmonella
case of hepatitis A virus (HAV) in- c. Shigellosis
fection in a 32-year-old man who d. Cryptosporidium
lives with his wife and one-year-old e. Campylobacter
twins. He is a self-employed con-
tractor who often eats on the run. 177. HSV-2 seroprevalence has
His wife works part-time at a book- increased by over 30% over the past
store and his children attend day two decades in the United States,
care. He has no history of travel, suggesting a continuing spread of
eating raw fish, or known contact herpes. Which of the following
with other cases of HAV infection. other epidemiologic findings have
been shown by recent studies?
174. The first step in investigating a. Only 50% of persons with HSV-2
this case is to confirm the diagnosis antibodies have been diagnosed
of HAV with with herpes
b. HVS-2 seropositivity correlates
a. A report of the history and exami- with viral shedding
nation from the treating physician c. Over 95% of genital infections are
b. Stool cultures caused by HSV-2
c. Total anti-HAV antibodies d. Recurrence rates for HSV-2 are the
d. IgM anti-HAV same as for HSV-1
e. HAV RNA e. Most transmissions occur during
the symptomatic phase
80 Preventive Medicine and Public Health

178. A wildlife worker presents to 179. The child is most at risk for
the emergency room because he which of the following complica-
was bitten on the hand by a raccoon tions?
while trying to capture the animal, a. Pneumonia
which appeared ill. He states he b. Reyes syndrome
received a primary course of rabies
c. Encephalitis
vaccination 11 2 years ago when he
first started his job. The wound is d. Orchitis
immediately thoroughly cleaned by e. Thrombocytopenia
the ER staff. It is small because he
was wearing gloves. Which is the 180. Which of the following is the
most appropriate intervention for most appropriate management of
rabies prevention? contacts?
a. No further prophylaxis is necessary a. Observation only for all contacts
because of the recent vaccination b. Vaccine for the mother, sibling, and
b. Administer rabies immune globulin susceptible classmates
(RIG) only c. Immune globulin for the mother,
c. Administer RIG and one dose of sibling, and susceptible classmates
vaccine d. Immune globulin for the mother
d. Administer one dose of vaccine and vaccine for his sibling and sus-
only ceptible classmates
e. Administer two doses of vaccine e. Immune globulin for the mother
and sibling, and vaccine for the
Items 179-180 susceptible classmates

A 5-year-old child presents to


the health department clinic with
fever, malaise, and a vesicular rash
that started 24 hours prior. He goes
to preschool. He has one sister aged
3 and his mother is 38 weeks preg-
nant. Both are susceptible.
Epidemiology and Prevention of Communicable Diseases 81

181. Consider the clinical presen- 182. Under which conditions


tation of the newborn in the follow- should chemoprophylaxis for influ-
ing figure. enza be considered?
a. All nursing home residents and
unvaccinated staff during an influ-
enza A outbreak
b. All nursing home residents and
unvaccinated staff during an influ-
enza B outbreak
c. Only unvaccinated nursing home
residents and staff during an influ-
enza A outbreak
d. Only unvaccinated nursing home
residents and staff during an influ-
enza B outbreak
e. All nursing home staff and residents
during an influenza B outbreak

183. For which patient is pneumo-


coccal vaccine PPV23 not benefi-
cial?
(Reproduced, with permission, from a. A 15-month-old HIV-infected child
Holmes KK, Sparling PF, Mardh P, et al.,
b. A 20-year-old about to undergo a
Sexually Transmitted Diseases, 3rd ed.,
splenectomy for ITP
New York, McGraw-Hill, 1999.)
c. A 70-year-old healthy female
d. A 5-year-old with sickle cell disease
This most likely represents congen- e. A 10-year-old with nephrotic syn-
drome who received the vaccine 5
ital
years ago
a. Rubella
b. Syphilis
c. Toxoplasmosis
d. Cytomegalovirus (CMV)
e. Varicella
82 Preventive Medicine and Public Health

184. Consider the epidemic curve illustrated in the following figure.

6 5

5 4

4 3 3 3 3 3

3 2 2 2
2 1 1 1 1 1 1 1

1 0 0 0 0 0 0 0 0 0
0
7/

7/

7/

7/

7/

7/

7/

7/

7/

7/

7/

7/

7/
4

2
0

Date of onset 8
(Source: Massachusetts Department of Public Health.)

The curve most likely represents a


a. Common-source outbreak epidemic curve
b. Propagated-source outbreak epidemic curve
c. Continual-source epidemic curve
d. Person-to-person outbreak epidemic curve
e. Point-source outbreak epidemic curve

185. A 10-month-old child is brought to your office by the mother


because of vomiting and profuse diarrhea for the last 24 hours. He has a
temperature of 100°F and has signs of dehydration. No other person in the
household is ill. The most likely etiologic agent responsible for the clinical
syndrome is
a. Adenovirus
b. Rotavirus
c. Parvovirus
d. Coxsackievirus
e. Echovirus
Epidemiology and Prevention of Communicable Diseases 83

186. The medical evaluation of a 188. Four-drug therapy is recom-


32-year-old HIV-infected patient mended as an initial approach to
reveals a tuberculin skin test reac- treatment for active TB in HIV-
tion at 5 mm. His chest x-ray is infected persons
normal. He is currently taking anti- a. Always
retroviral therapy which includes b. When multidrug-resistant TB ex-
protease inhibitors. He has not pre- ceeds 4% in the community
viously received therapy for tuber- c. When the patient has had previous
culosis in the past nor has he had therapy for TB
any known contact with persons d. When the patient has had a known
infected with tuberculosis. Which exposure to multi-drug resistant TB
e. When the CD4 cell count is under
is the most appropriate interven-
200
tion for this patient?
a. No preventive therapy for tubercu- 189. You are contacted by a local
losis
physician who wishes to inform you
b. Izoniazid for nine months
c. Rifampin for nine months
that she diagnosed and confirmed a
d. Rifampin and pyrazinamide for two case of hepatitis A in one of her
months patients, a 5-year-old who attends a
e. Streptomycin for six months preschool center. She is concerned
about the staff and children attend-
187. HIV-infected persons are at ing the school center. Which is the
highest risk of having an active TB most appropriate management of
infection resistant to susceptible contacts?
a. Izoniazid a. Immune globulin to all staff and
b. Rifampin children
c. Streptomycin b. Vaccine to all staff and children
d. Ethambutol c. Vaccine to staff and immune globu-
e. Pyrazinamide lin to all children
d. Immune globulin and vaccine to
staff and all children
e. Immune globulin only to classroom
contacts
84 Preventive Medicine and Public Health

190. A 22-year-old woman pre- 191. Which persons in your city


sents to the obstetrical clinic for her are at highest risk of developing
second prenatal visit. She is 28 severe infection?
weeks pregnant. The examination is a. The elderly
normal. She reports multiple sexual b. Newborns and young children
partners, but denies drug use. The c. Diabetics
RPR done at the first prenatal visit 8 d. HIV-infected persons
weeks ago was 1:64 with a positive e. Pregnant women
TP-PA (treponemal test). The clinic
was unable to reach her. She does 192. What public health advisory
not recall ever being treated for measure would you announce to
syphilis in the past, nor does she prevent ingestion of contaminated
remember any symptoms. The rapid water?
RPR card test done at this visit is a. Drink bottled water only
positive. She is allergic to penicillin. b. Use faucet filters capable of remov-
The most appropriate intervention ing particles of 2.0 microns
is to c. Boil water for 1 minute
d. Disinfect with chlorination
a. Send for an RPR titer and trepone-
e. Freeze and use thawed water
mal test to determine appropriate
treatment
b. Treat with erythromycin 193. As medical director of a health
c. Treat with doxycycline maintenance organization, you are
d. Admit for desensitization and treat asked to update screening recom-
with penicillin mendations for enrolled members.
e. Treat with ceftriaxone You find that recommendations are
lacking in the field of sexually
Items 191192 transmitted diseases. You decide to
develop evidence-based screening
Following multiple reports of
guidelines for Chlamydia trachomatis.
cases of Cryptosporidium parvum di-
Which of the following criteria is the
agnosed by private physicians, as
most important for developing rou-
medical director of City XYZ Health
tine screening recommendations?
Department, you conduct an epi-
demic investigation leading to the a. Number of sexual partners
b. Use of barrier methods such as con-
conclusion that the city drinking
doms
water supply is contaminated with c. Contact with an infected person
C. parvum. d. Presence of symptoms
e. Age
Epidemiology and Prevention of Communicable Diseases 85

194. A 20-year-old male presents with complaints of dysuria and urethral


discharge for three days. He engaged in unprotected vaginal intercourse 8
days ago with a new female sexual partner. She has no complaints. Exami-
nation reveals a yellow urethral discharge. The gram stain is as follows:

(Reproduced, with permission, from Holmes KK, Sparling PF, Mardh P,


et al., Sexually Transmitted Diseases, 3rd ed., New York, McGraw-Hill,
1999.)

For which of the following organisms can a presumptive diagnosis be


made:
a. Chlamydia trachomatis
b. Treponema pallidum
c. Ureaplasma unrealyticum
d. Neisseria gonorrhoeae
e. Herpes simplex virus infection
86 Preventive Medicine and Public Health

195. A 30-year-old Canadian im- 197. A 20-year-old college student


migrant farmer consults with symp- presents to your office because she
toms of night sweats, low-grade noticed two bumps on her vulva 1
fever, cough, and fatigue. He does week ago. She does not complain
not smoke. He has a history of about pain or discharge. She has
asthma. The chest x-ray required been sexually active with the same
for immigration was normal five partner for one year. He has no
months ago. He received the BCG symptoms. She has noticed one
vaccine as a child. The skin test for similar lesion on his penis. Upon
tuberculosis is positive at 15 mm. examination, you notice two condy-
The most likely diagnosis is lomata acuminata of 0.5 cm in size
a. Influenza at the fourchette. Which of the fol-
b. Brucellosis lowing counseling messages is the
c. Aspergillosis most appropriate?
d. Mycobacterium bovis a. Treatment of her sexual partner will
e. Mycobacterium tuberculosis reduce the risk of recurrence of her
vulvar lesions
196. A healthy 2-month-old infant b. Treatment of her vulvar lesions will
is brought to the office for routine reduce her risk of developing cervi-
child care. The child has a normal cal cancer
growth curve. She received the first c. Condom use is very effective in
dose of hepatitis B vaccine at birth reducing transmission of this infec-
as well as a dose of HBIG because tion
d. Recurrence of lesions is more fre-
the mother was HBsAg-positive.
quent in the first year after initial
Which of the following vaccine diagnosis
series should be administered at e. Pap smear screening should be per-
this time? formed every six months
a. MMR, OPV, DTP, Hep B
b. IPV, Hib, DTP, Hep B
c. Hep B, DTaP, Hib, IPV
d. DTaP, Hib, IPV
e. IPV, DTaP, Hep B
Epidemiology and Prevention of Communicable Diseases 87

DIRECTIONS: Each group of questions below consists of lettered


options followed by numbered items. For each numbered item, select the
appropriate lettered option(s). Each lettered option may be used once,
more than once, or not at all. Choose exactly the number of options indi-
cated following each item.

Items 198-200 Items 201-204


Match each group of diseases Various terms and parameters
with the most common described are used in epidemiological studies
mode of transmission. of infectious diseases. Match each
a. Water- or foodborne transmission statement below with the most ap-
b. Zoonoses propriate descriptive term.
c. Person-to-person direct contact a. Immunogenicity
transmission b. Pathogenicity
d. Airborne transmission c. Infectivity
e. Arthropod-borne transmission d. Virulence
f. Sexual transmission e. Incubation

zoonoses 198. Rabies, psittacosis, salmonel- 201. Neutralizing antibody devel-


losis. (SELECT 1 DESCRIPTION) ops in 95% of people after an attack
of measles. (SELECT 1 TERM)
airborne 199. Measles, tuberculosis, influ- immunogenicity
transmission enza. (SELECT 1 DESCRIPTION) 202. Febrile respiratory tract dis-
ease develops in approximately 80%
200. Cyclospora, Campylobacter, Yer- of children infected with influenza.
sinia. (SELECT 1 DESCRIPTION) (SELECT 1 TERM) pathogenicity
water or foodborne
transmission 203. Death occurs in approxi-
mately 20% of cases of pneumococ-
cal meningitis. (SELECT 1 TERM)
virulence
204. Approximately 50% of house-
hold contacts of a child who has a
common cold become infected. (SE-
LECT 1 TERM) infectivity
88 Preventive Medicine and Public Health

Items 205-207
Consider the epidemiologic curves of sexually transmitted diseases
reported to the CDC in the United States over the last 15 years. Match each
curve with the appropriate infection.
a. Chlamydia
b. Gonorrhea
c. Syphilis
d. Herpes
e. Trichomonas
f. Human papillomavirus infection
g. Hepatitis B

400
350

300
Rate per 100,000

250

200
1 chlamydia
150

100 2 gonorrhea

50
3 syphilis
84 85 86 87 88 89 90 91 92 93 94 95 96 97 98
Year

205. Curve 1

206. Curve 2

207. Curve 3
Epidemiology and Prevention of Communicable Diseases 89

Items 208-210 Items 211-213


Choose the most likely infec- Match each of the statements
tious agent for each description of below with the hepatitis virus with
the events following consumption which it has been most closely
of food. associated.
a. Staphylococcal enterotoxin a. Hepatitis A virus (HAV)
b. Clostridium botulinum toxin b. Hepatitis B virus (HBV)
c. Enterotoxic Escherichia coli c. Hepatitis C virus (HCV)
d. Clostridium perfringens d. Hepatitis D virus (HDV)
e. Salmonella typhimurium e. Hepatitis E virus (HEV)
f. Giardia lamblia
g. Crytosporidium 211. Case fatality rate can be as
h. Campylobacter high as 20% if acute infection
occurs during the third trimester of
208. Within 4 h after attending a pregnancy. (SELECT 1 VIRUS) HEV
church supper, 25 persons report
the abrupt onset of nausea, vomit- 212. Coinfection must exist for
ing, and diarrhea. (SELECT 1 replication and infection to occur.
AGENT) Staph enterotoxin (SELECT 1 VIRUS) HDV

209. One week after arriving in 213. Chronic disease develops in


Africa, 16 students develop vomit- over 50% of persons following an
ing, severe diarrhea, and abdomi- acute infection. (SELECT 1
nal cramps lasting 2 to 3 days. VIRUS) HCV
(SELECT 1 AGENT) ETEC
Items 214-216
210. One-third of the persons who
attended a school banquet develop Match each of the diseases be-
abdominal cramps and watery diar- low with the appropriate epidemio-
rhea 8 to 12 h later. These symp- logic term.
toms end within 24 h. (SELECT 1 a. Hyperendemic
AGENT) b. Epidemic
c. Endemic
Clostridium perfringens d. Enzootic
e. Pandemic
f. Epizootic
90 Preventive Medicine and Public Health

endemic 214. Lyme disease in the 1990s. Items 220-224


Select the reservoir for each of
epidemic 215. Cholera among Rwandan ref- the diseases below.
ugees in 1994.
a. Cattle
b. Humans
pandemic 216. Influenza in 1919. c. Rodents
d. Ticks
Items 217-219 e. Mosquitoes
Match each infection below f. Cats
with the intermediate host involved g. Soil
h. Vegetation
in transmission.
a. Snail
220. Nocardiosis. (SELECT 1 soil
b. Swine
RESERVOIR)
c. Fish
d. Crab
e. Dog 221. Hantavirus. (SELECT 1 deer mouse
f. Cattle RESERVOIR)
g. Deer
h. Sheep 222. Brucellosis. (SELECT 1 cattle
RESERVOIR)
crab 217. Paragonimiasis (lung fluke 223. Enterobiasis. (SELECT 1 humans
disease). (SELECT 1 HOST)
RESERVOIR)
dog 218. Toxocariasis (visceral larva
migrans). (SELECT 1 HOST) 224. Toxoplasmosis. (SELECT 1 cats
RESERVOIR)
swine 219. Cysticercosis. (SELECT 1
HOST) Items 225227
For each disease, choose the
most effective or principal means of
control.
a. Rat control
b. Sanitation
c. Immunization
d. Vector control
e. Deer control
Epidemiology and Prevention of Communicable Diseases 91

vector 225. St. Louis encephalitis. (SE- 229. A 4-year-old girl presents
control LECT 1 CONTROL) with sore throat, fever, hoarse-
ness, and drooling. (SELECT 1
sanitation 226. Typhoid fever. (SELECT 1 AGENT) Hib
CONTROL)
230. A 35-year-old woman pre-
227. Tetanus. (SELECT 1 CON- sents with painful muscular con-
immunization TROL) tractions of the masseter and neck
muscles. (SELECT 1 AGENT)
Items 228230 Clostridium tetani
Items 231-232
Match each of the descriptions
below with the correct etiologic For each dose schedule, select
agent. the appropriate vaccine.
a. Clostridium botulinum a. Pneumococcal vaccine
b. Clostridium tetani b. Oral polio vaccine (OPV)
c. Poliovirus c. Inactivated polio vaccine (IPV)
d. Corynebacterium diphtheriae d. Varicella vaccine
e. Haemophilus influenza B e. Measles-mumps-rubella vaccine
f. Borrelia burgdorferi (MMR)
f. Influenza virus vaccine
228. A 25-year-old man presents
with blurred vision, dysphagia, and 231. Recommended for the first
IPV
dry mouth. (SELECT 1 AGENT) two doses at 2 and 4 months of age.
Clostridium botulinum 232. Second dose recommended
at age 4 to 6 years. MMR
ENVIRONMENTAL AND
OCCUPATIONAL HEALTH
Questions
DIRECTIONS: Each item below contains a question or incomplete
statement followed by suggested responses. Select the one best response to
each question.

233. Which etiological agent was 235. Following an accident in a


responsible for most cases of illness nuclear laboratory, some workers
due to waterborne-disease outbreaks were exposed to 300 rem (3 Sievert)
in the United States in the 1990s? of radiation. They are immediately
a. Salmonella enteritidis (serotype ty- sent to your emergency department.
phimurium) Which of the following effects will
b. Giardia lamblia most likely occur among the major-
c. Campylobacter jejuni ity of these workers?
d. Cryptosporidium parvum
a. Bone marrow depression
e. Shigella sonnei
b. Neurovascular syndrome
c. Gastrointestinal syndrome
234. The most important risk fac- d. Cardiovascular syndrome
tor for heat-related illness is e. No detectable physiological effect
a. Age over 65
b. Age under 1 236. The Haddon matrix is used
c. History of prior heat stroke for assessing interventions for the
d. Low socioeconomic status prevention of
e. Obesity
a. Water pollution
b. Air pollution
c. Radiation exposure
d. Injury
e. Toxic substance exposure

119

Terms of Use
120 Preventive Medicine and Public Health

237. Which of the following pure tone audiograms best represents mild
noise-induced hearing loss?

B
Environmental and Occupational Health 121

D
122 Preventive Medicine and Public Health

E
(Audiograms reproduced, with permission, from LaDou J., Occupational and Envi-
ronmental Medicine, 2nd ed., Stamford, CT,Appleton & Lange, 1997: pp 125129.)

238. The most effective means of preventing trichinosis in humans is


a. Cooking pork to reach a internal temperature of at least 40°C (104°F)
b. Proper disposal of hog feces
c. Prohibiting feeding garbage to hogs
d. Testing hogs with Trichinella antigen prior to slaughter
e. Freezing pork at 10°F

239. Which engineered water purification system is the most effective for
the elimination of Cryptosporidium parvum?
a. Flocculation
b. Sedimentation
c. Disinfection
d. Boiling
e. Filtration
Environmental and Occupational Health 123

240. The major environmental Items 242-244


source of lead absorbed in the hu-
A 42-year-old welder is brought
man blood stream in adults is
in the emergency room complain-
a. Air ing of a sore throat, headache, and
b. Water
myalgias. He also started feeling a
c. Lead-based paint
tightness in the chest and shortness
d. Food
e. Soil of breath. He works in an electro-
plating operation brazing and cut-
241. You are asked to evaluate the ting metals. Pulmonary function tests
working environment in a manufac- reveal a reduced forced expiratory
turing plant processing metal parts. volume. The chest x-ray is normal.
In one area of the mill, where such
parts are flattened, the sound level is 242. Which of the following expo-
measured at 85 dB. The workers sures is the most likely cause of the
responsible for this process are ex- workers symptoms?
posed to this sound for the entire a. Lead
8-hour shift. The most appropriate b. Mercury
intervention for this level of sound is c. Chromium
d. Copper
a. None. This level of sound is below e. Cadmium
the level at which OSHA regula-
tions apply
243. The most likely source of ab-
b. A hearing conservation program
c. A shutdown of the manufacture sorption is
until the level of sound is reduced a. Lung
d. A shutdown only of the process area b. Skin
where the sound is 85 dB or higher c. Mucous membranes
e. Enforcement of hearing protective d. Gastrointestinal
devices for all exposed workers e. Open sores

244. Which of the following should


be used to treat acute exposure?
a. EDTA
b. Pralidoxime
c. Dimercaprol
d. Acetylcysteine
e. Atropine
124 Preventive Medicine and Public Health

245. Toxicology is the study of ad- 247. She most likely suffers from
verse effects of chemicals on living a. Sunstroke
organisms. Which of the following b. Heat cramps
occurrences would be indicative of c. Heat exhaustion
the most important nonthreshold d. Heat stroke
effect in humans? e. Heat syncope
a. Infertility
b. Paralysis 248. The most appropriate cool-
c. Adenocarcinoma ing measure for this patient is
d. Neutropenia a. Immersion in ice-water bath
e. Cirrhosis b. Iced gastric lavage
c. Ice packs to groin, axilla, and neck
246. What proportion of cancers d. Evaporative cooling
in humans is estimated to be the re- e. Cool and shaded environment
sult of environmental factors?
a. 10% 249. In addition to proper hydra-
b. 25% tion, rest, and attention to heat index
c. 50% guidelines, she should be advised to
d. 75% avoid reexposure to heat for at least
e. 90% a. 1 day
b. 1 week
Items 247-249 c. 2 weeks
d. 3 weeks
A 34-year-old woman is e. 4 weeks
brought in from a sporting event
complaining of headache, nausea, 250. Which of the following tests
and weakness. She had been jog- is the most frequently used rapid
ging outside in sunny weather screening test to assess mutagenic-
where the temperature was 90° ity/carcinogenicity of a chemical
Fahrenheit with a relative humidity substance?
of 70%. She had started a training
a. Ames test
program two weeks before. She is
b. Mammalian mutation assay
hyperventilating, her skin is moist, c. Unscheduled DNA assay
and her core body temperature is d. Cell transformation assay
38.8° Celsius. e. Cytogenetic assay
Environmental and Occupational Health 125

251. A migrant farm worker is 254. Radioactive waste is best dis-


brought to the clinic at 2:00 P.M. posed by
complaining of blurred vision, sali- a. Physical treatment
vation, nausea, and diarrhea. He had b. Incineration
been working in the fields since 6:00 c. Landfill
A.M. in hot and humid weather. The d. Injection wells
examination reveals the following e. Chemical treatment
findings: heart rate of 50 per minute,
respiration 20 per minute, profuse 255. Which of the following is re-
perspiration, and miosis. The most sponsible for the largest proportion
effective initial intervention with this of domestic water use?
worker is a. Bathing
a. Rapid administration of intravenous b. Drinking
fluid c. Laundry
b. Evaporative cooling d. Toilet flushing
c. Atropine e. Dishwashing
d. Observation only
e. Epinephrine 256. On a hot summer day in a
large urban center located in the
252. Which of the following waste southwestern United States, an
management methods is the pre- emergency room department re-
ferred method of waste control? ports an increase in admissions for
a. Waste minimization asthma in children and young
b. Incineration adults, but not among patients suf-
c. Recycling fering from chronic bronchitis or
d. Physical treatment ischemic heart disease. The most
e. Biological treatment likely air pollutant responsible for
the exacerbation of asthma is
253. Commercial airline pilots have a. CO
higher exposures to which type of b. Ozone
radiation compared to the general c. Nitrogen dioxide
population? d. Particulate matter
a. Alpha particles e. Lead
b. Beta particles
c. Gamma rays
d. Cosmic rays
e. X-rays
126 Preventive Medicine and Public Health

257. A 42-year-old welder pre- 261. A 28-year-old woman pre-


sents to employee health services sents with nausea, vomiting, and di-
complaining of tearing eye pain and arrhea. She has no fever. Her history
photophobia. A photokeratocon- reveals that she attended a reception
junctivitis is diagnosed. The most about six hours ago. She ate roast
likely cause of this condition is beef with gravy, salad, and had
a. Infrared radiation cream-filled pastries for dessert. Pre-
b. Visible radiation vention of this food-borne illness
c. Ultraviolet radiation A could have been achieved by
d. Magnetic radiation a. Freezing the food
e. Ultraviolet radiation B b. Heating the food to 140° Fahren-
heit
258. Which of the following phys- c. Proper hand washing by food han-
ical characteristics of water is the dlers
most important impediment to dis- d. Proper cleaning of contaminated
infection? surfaces
e. Control of flies
a. Color
b. Viscosity
c. Turbidity 262. A 50-year-old textile worker
d. Density presents to your office for his peri-
e. Temperature odic health examination. He has
no complaints. Review of history
259. Which of the following resi- reveals that he has been working
dential environmental pollutants is for over 25 years at the same com-
the leading cause of lung cancer? pany. His work consists of prepar-
a. Radon ing dyes. Which of the following
b. Tobacco smoke tests would be appropriate in this
c. Asbestos setting?
d. Formaldehyde a. A chest x-ray
e. Sulfur oxide b. A brain computed tomography
(CT) scan
260. Which of the following min- c. Liver function tests
erals is responsible for har d water? d. A complete blood count
a. Lead e. A urinalysis
b. Copper
c. Iron
d. Sulfur
e. Manganese
Environmental and Occupational Health 127

263. A 30-year-old patient presents at an evening walk-in clinic after work


complaining of chills, fever, and malaise of acute onset. He started coughing
and feeling out of breath late in the afternoon. Inspiratory crackles are present
on chest auscultation. The chest x-ray is normal. The complete blood count
reveals 12,000 WBC with 70% PMNs. His past medical history is benign. No
one else in the household is sick. He says some of his coworkers have a cold.
He works in a pet shop in the bird section. He is not taking any medication.
He states he had a similar episode a few weeks ago that resolved after a few
days of rest at home. The most appropriate management is to prescribe
a. Rest, fluid, and antipyretics
b. Amantadine
c. Doxycycline
d. Prednisone
e. Erythromycin

Items 264-265
A 45-year-old quarry worker presents with a history of progressive
nonproductive cough and dyspnea. He has no fever or weight loss. The
complete blood count is normal. The chest x-ray is as follows.

(Reproduced, with permission, from LaDou J., Occupational


and Environmental Medicine, 2nd ed., Stamford, CT,Apple-
ton & Lange, 1997: 320.)
128 Preventive Medicine and Public Health

264. The most likely diagnosis is 268. Which of the following smok-
a. Caplan s syndrome ers has the highest risk of develop-
b. Silicosis ing lung cancer?
c. Asbestosis a. An asbestos worker
d. Byssinosis b. A uranium miner
e. Sarcoidosis c. A coal worker
d. A textile worker
265. He is at highest risk of devel- e. A sandblaster
oping
a. Lung cancer Items 269-270
b. Tuberculosis
A 30-year-old man has been
c. Cryptococcosis
d. Chronic bronchitis
planning a two-week mountain-
e. Emphysema climbing trip with three other col-
leagues. He is in good health. He
266. Biological oxygen demand has been exercising regularly for
(BOD) measures the total organic many months in anticipation of the
content of water based on the con- trip. They will be climbing to a
sumption of oxygen in a sample at maximum altitude of 8,500 feet. He
20° Celsius over five days. A con- is planning on leaving the next day.
sumption of 10 to 20 mg of O2 per
liter most likely represents a sample 269. When reaching the summit,
from this patient is at highest risk of devel-
oping
a. Treated freshwater
b. Untreated freshwater a. Headache, nausea, and sleep dis-
c. Treated sewage turbances
d. Domestic sewage b. Cough, tachypnea, and rales
e. Industrial wastewater c. Headache, ataxia, and altered men-
tal status
d. Abdominal pain, flatulence, and di-
267. Which of the following fac-
arrhea
tors of air travel is most likely to e. Leg pain and swelling
adversely impact a patient with car-
diopulmonary disease?
a. Immobility
b. Cabin air quality
c. Barometric pressure
d. Temperature
e. Humidity
Environmental and Occupational Health 129

270. In addition to recommending 274. Following a boating accident


a slow ascent, prophylaxis for this at sea, victims are rescued by the
patient could include Coast Guard and are immediately
a. Nifedipine rushed for emergency medical care.
b. Furosemide Persons who were rescued from the
c. Acetazolamide water are more likely to experience
d. Dexamethasone hypothermia than those exposed
f. Erythropoietine only to cold air. The most likely
mechanism is
271. The most important risk fac- a. Vasodilation
tor for motor vehicle injury is b. Thermal conductivity
a. High speed c. Loss of protective barriers
b. Lack of use of vehicle restraints d. Head injury
c. Driving at night e. Exhaustion from efforts to stay afloat
d. Decreased vehicle size
e. Alcohol ingestion 275. During a diving expedition to
explore sunken ships, one of the
272. Which of the following dis- divers starts to experience light-
eases is found almost exclusively headedness, dizziness, ataxia, and
among persons who have worked nausea after reaching 110 feet in
with or have been exposed to depth. Which of the following is
asbestos? the most likely diagnosis?
a. Bronchogenic carcinoma a. Nitrogen narcosis
b. Byssinosis b. Barotrauma
c. Pleural mesothelioma c. Vertigo
d. Laryngeal carcinoma d. Barosinusitis
e. Emphysema e. Bends

273. The industry that has the 276. Which of the following sub-
highest accidental death rate in the stances is causally associated with
United States is pneumoconiosis?
a. Manufacturing a. Sulfur oxides
b. Construction b. Nitrogen oxides
c. Mining and quarrying c. Oil fumes
d. Transportation and public utilities d. Dust particles
e. Service e. Cigarette smoke
130 Preventive Medicine and Public Health

277. Some agents have been found 279. Different reproductive out-
to be neurotoxic to the fetus and af- comes can be used in studies exam-
fect pregnancy outcomes. At which ining the effect of exposure to a
of the following periods will the particular potentially toxic sub-
fetus be particularly susceptible to stance. Which of the following stud-
neurotoxic substances? ies is most likely to be subject to
a. 3 to 16 weeks bias?
b. 6 to 9 weeks a. A study examining an association
c. 4 to 8 weeks with early spontaneous abortion
d. 3 to 6 weeks b. A study examining an association
e. 7 to 9 weeks with late spontaneous abortion
c. A study examining an association
278. A couple presents to the in- with congenital anomalies
fertility clinic because of inability to d. A study examining an association
conceive for over one year. A semen with preterm birth
analysis on the male reveals oligo- e. A study examining an association
with low birth weight
spermia. He works for a company
that manufactures storage batteries.
A blood level should be obtained 280. A large explosion occurs at a
for which of the following agents? construction site during excava-
tion. None of the workers appear
a. Chromium
injured. Some of them were ex-
b. Nickel
c. Lead
posed to sound pressure levels of
d. Antimony 190 dB. Which of the following is
e. Boron the most likely outcome for these
workers?
a. Temporary tinnitus
b. Temporary conductive hearing loss
c. Permanent conductive hearing loss
d. Temporary sensorineural loss
e. Permanent sensorineural loss
Environmental and Occupational Health 131

281. Ergonomics is also called hu- 284. A 50-year-old presents with


man factors engineering, and ex- dyspnea on exertion, without cough
amines ways to adapt the working or chest pain. He has no history of
environment to ensure a safe and asthma, chronic bronchitis, or heart
productive workplace. Which of the disease. He does not smoke. He is
following factors is the most impor- employed in the aircraft industry
tant to improve the physical design and his work consists of producing
of a sedentary job? metal alloys. His chest x-ray reveals
a. Maintaining a static position small, rounded, and irregular opaci-
b. Maintaining a standing position ties. Pulmonary function tests show
c. Eliminating the waist motion decreased diffusion. The Kveim for
d. Installing a soft floor sarcoidosis is negative. The most
e. Maintaining a static holding posi- likely etiologic agent responsible for
tion these findings is
a. X-rays
282. Which of the following meth- b. Beryllium
ods is most effective in reducing c. Tantalum
radon levels in homes and build- d. Uranium
ings? e. Carbon dioxide
a. Maintaining a sealed environment
and recirculating air 285. Vibration, low temperatures,
b. Repairing cracks in the foundation repetition, and force can all con-
c. Keeping windows open tribute to the development of re-
d. Venting air on the upper floors petitive motion disorders. Which of
e. Insulating the basement the following industries is associated
with the highest rate of disorders
283. Har d water has been asso- associated with repeated trauma?
ciated with which of the following
a. Grocery stores
beneficial health effects? b. Manufacturing electronic equip-
a. Decrease in cardiovascular disease ment
b. Decrease in colorectal cancer c. Computer manufacturing
c. Decrease in lung cancer d. Meat-packing plants
d. Decrease in anemia e. Poultry slaughtering
e. Decrease in osteoporosis
132 Preventive Medicine and Public Health

286. You are employed by a city 289. Hazardous waste site cleanup.
health department and oversee the B
quality of recreational waters in Items 290-293
your area. There is a lake with a
Match the following clinical
beach that is very crowded during
presentations with the most likely
the summer. Which of the follow-
metal exposure.
ing organisms would you quantita-
tively measure on a regular basis to a. Arsenic
b. Beryllium
assess the safety of the water?
c. Cadmium
a. Coliform d. Chromium
b. Escherichia coli e. Lead
c. Giardia lamblia f. Manganese
d. Norwalk virus g. Mercury
e. Salmonella h. Nickel
i. Zinc
Items 287-289
Match the following events with 290. A worker presents with hy-
the most appropriate legislative act. perkeratosis, hyperpigmentation,
and anemia. arsenic
a. Medical Waste Tracking Act 1988
b. Comprehensive Environmental Re-
sponse, Compensation and Liabili- 291. A worker presents with Fan-
ties Act 1980 conis syndrome. cadmium
c. Resource Conservation and Recov-
ery Act 1976 292. A worker experiences fever,
d. National Environmental Protection chills, profuse sweating, cough,
Act 1970 and chest pain that resolves after 48
e. Federal Insecticide, Fungicide and hours. zinc
Rodenticide Act 1972
f. Toxic Substance Control Act 1976 293. A worker presents with ataxia,
g. Clean Water Act 1972
loss of visual fields, and auditory
h. Safe Drinking Water Act 1974
i. Clean Air Act 1970
disturbances. mercury

287. Ban of the manufacturing


and distribution of asbestos.
F
288. Ban of the use of PCBs for all
but emergencies. E
Environmental and Occupational Health 133

Items 294-295 298. Chloracne (skin). DDT


Match the following clinical
Items 299-300
presentation with the most likely
solvant exposure. For each poisoning with the
a. Hydrocarbons agents listed below, select the ap-
b. Petroleum distillates propriate treatment.
c. Alcohols a. Pralidoxime
d. Glycols b. Amyl nitrite
e. Ketones c. Dimercaprol
f. Esters d. Edetate calcium disodium
g. Phenols e. Acetylcysteine
f. Flumazenil
294. A worker presents with op-
tic neuropathy, blurred vision, and 299. Parathion. Pralidoxime
blindness. alcohol
300. Mercury. Dimercaprol
295. A worker presents with he-
patic and kidney necrosis. Items 301-303
phenols Match each of the workers be-
Items 296-298
low with the infectious disease for
Match the following organ tox- which they are at risk.
icity with the most likely exposure. a. Hepatitis B
a. Arsenic b. Brucellosis
b. Carbon tetrachloride c. Legionnaires disease
c. Quartz d. Histoplasmosis
d. Coal e. Sporotrochosis
e. Cotton
f.
g.
Acrylic
DDT
301. Butcher. Brucellosis
302. Air conditioner repair person.
296. Cardiovascular toxicity, in- Legionnaires
cluding arrythmia, myocardial in- 303. Dentist.
jury, and peripheral arterial occlusive
disease. arsenic Hepa B
297. Acute liver toxicity with ne-
crosis (liver).
carbon tetrachloride
134 Preventive Medicine and Public Health

Items 304-305 304. Benzene. hematopoietic


Certain substances in the occu-
pational environment have been
305. Radium. bone
identified as carcinogenic agents
based on epidemiologic evidence
obtained in studies of exposed labo-
ratory animals and human popula-
tions. Match each chemical agent
with the human target site for cancer.
a. Liver
b. Brain
c. Bladder
d. Lung
e. Hematopoietic systems
f. Bone
EPIDEMIOLOGY
AND PREVENTION
OF NONCOMMUNICABLE
AND C HRONIC DISORDERS
Questions
DIRECTIONS: Each item below contains a question or an incomplete
statement followed by suggested responses. Select the one best response to
each question.

306. The American Academy of 307. For which of the following


Pediatrics (AAP) revised its recom- conditions is this child at highest
mendations for fluoride supple- risk?
ments in 1995. What fluoride a. Decreased intelligence test scores
supplement would you recommend b. Impaired growth
for a 4-year-old child if the water c. High blood pressure
level in the community where she d. Chronic renal disease
lives is 0.3 parts per million? e. Hepatic toxicity
a. No supplement
b. 0.25 mg per day 308. Which intervention is the
c. 0.50 mg per day most important for this child?
d. 0.75 mg per day a. Treatment with iron supplements
e. 1.00 mg per day b. Chelation therapy with
d-penicillamine
Items 307-308 c. Chelation therapy with EDTA
d. Elimination of lead in the child s en-
An asymptomatic 2-year-old vironment
child living in a delapidated older e. Treatment with calcium supple-
building in an inner-city neighbor- ments
hood is screened for elevated lead
levels. The results show a blood
lead level of 30 µg/dL.

151

Terms of Use
152 Preventive Medicine and Public Health

309. A 25-year-old woman wants 310. What is the most appropriate


to lose weight before going on a trip approach to promote smoking ces-
to the Caribbean. She has joined a sation for this patient?
health club and signed up for classes a. Refer him to classes for smoking
with a stationary bike in which each cessation and reassess progress in 2
40-min session burns up 500 calo- weeks
ries. She is taking two sessions a b. Provide self-help materials and
week. Assuming her caloric intake reassess in 3 months
remains the same, how many weeks c. Prescribe nicotine replacement ther-
will it take to lose 6 pounds? apy and reassess progress in 2 weeks
d. Set a quit date with the patient and
a. 5 reassess his situation 2 days after
b. 9 this date
c. 15 e. Give clear, personalized advice to
d. 21 quit and readdress the issue at the
e. 27 next visit

Items 310-312 311. His blood cholesterol is re-


A 50-year-old man presents to a peated, and the results are the
health center for routine care. His same. Which of the following is the
last visit was 5 years ago and he has most appropriate intervention for
no complaints. He has been smok- his lipid profile?
ing 1 pack of cigarettes a day since a. Repeat blood cholesterol in 1 year;
he was 15 years old. When coun- no therapy is indicated
seled about his smoking, he says he b. Recommend one alcoholic drink
has no intention of quitting and feels per day
fine. He drinks two alcoholic bever- c. Recommend dietary therapy
d. Recommend dietary and drug ther-
ages per week. Records show that
apy
his blood cholesterol is 235 mg/dL, e. Recommend dietary therapy; if in-
with an HDL level of 40 mg/dL and effective, add drug therapy
an LDL level of 140 mg/dL. He has
no family history of coronary artery
disease (CAD). His height is 5′10″
and he weighs 170 lbs. His blood
pressure is 110/75 mm Hg.
Noncommunicable and Chronic Disorders 153

312. According to the U.S. Pre- 315. A 16-year-old boy is diag-


ventive Services Task Force, which nosed with depression following the
additional preventive health mea- divorce of his parents. He suffers
sure is indicated? from lack of appetite, insomnia,
a. Fecal occult blood testing (FOBT) feelings of worthlessness, and diffi-
b. Influenza vaccine culty in concentrating. He is given
c. Chest x-ray antidepressants and is referred to a
d. Prostate-specific antigen (PSA) psychologist for weekly psychother-
e. Fasting blood glucose apy visits. Which of the following is
the most important risk factor for
313. Which of the following inter- committing suicide?
ventions is the most effective in the a. Social isolation
prevention and control of injuries? b. Access to lethal medication
a. Education c. Noncompliance with antidepres-
b. Economic incentives sant medication
c. Law enforcement d. Access to firearms
d. Engineering e. Alcohol abuse
e. Emergency response
316. The most important cause of
314. Screening to detect problem years of potential life lost in the
drinking is recommended for all ages United States is
by the U.S. Preventive Services Task a. Cancer
Force. The most effective method for b. Cardiovascular disease (CVD)
early detection of alcohol abuse is c. HIV infection
a. Liver function tests d. Injuries
b. Blood alcohol level e. Perinatal mortality
c. Questioning the family
d. Asking the patient about the quan- 317. In most states, the legal limit
tity and frequency of alcohol use for blood alcohol concentration
e. Using a standardized questionnaire allowed while operating a motor
vehicle is
a. 1020 mg/dL
b. 4060 mg/dL
c. 80100 mg/dL
d. 120140 mg/dL
e. 150200 mg/dL
154 Preventive Medicine and Public Health

318. An elderly homeless man in 320. What proportion of the U.S.


brought in by the police on a win- adult population is estimated to
ter night because he was found have a mental or emotional prob-
wandering the streets confused and lem that requires therapy?
hallucinating. His consciousness is a. 1%
dulled and the ECG shows the fol- b. 5%
lowing findings: c. 15%
d. 25%
e. 40%
VS
321. In country A, there are 35 new
cases of breast cancer per 100,000
adult women per year. In country B,
(Adapted, with permission, from Fauci
the number is 90 per 100,000.
AS, Braunwald E, Isselbacher KJ, eds.,
Harrison s Principles of Internal Medicine, Which of the following is the most
14th ed., New York, NY, McGraw-Hill, likely explanation?
1998: 1, 246.) a. Women in country A have a much
higher rate of nursing their infants
b. Women in country A are less likely
The most likely cause of these find- to smoke cigarettes
ings is c. Women in country A receive more
a. Hypothermia frequent preventive care, such as
b. Hypoglycemia mammography
c. Acute alcohol intoxication d. Treatment is much more successful
d. Dementia in country A
e. Sepsis e. Women in country A are younger

319. A 27-year-old pregnant


woman is brought to the emergency
room with multiple ecchymoses to
the chest and abdomen. Her breath
smells of alcohol. The most likely
cause of these findings is
a. Hepatic failure
b. Domestic violence
c. An accidental fall
d. An automobile accident
e. Disseminated intravascular coagu-
lation
Noncommunicable and Chronic Disorders 155

Items 322-323 324. A 52-year-old woman pre-


sents to your office for her annual
The overall infant mortality
gynecological examination. She
rate (IMR) has declined in the
stopped menstruating about 6
United States since the beginning
months ago and is getting some hot
of the century. In some U.S. cities
flashes. Her history reveals that she
in 1900, up to 30% of infants
drinks one glass of wine per day
would die before reaching the age
and smokes about 10 cigarettes per
of one. Overall rates have dropped
day. She does not exercise much
from over 800 per 100,000 to less
and is overweight. Her most impor-
than 10 per 100,000 in 1998.
tant risk factor for developing os-
teoporosis is
322. Which of the following is the
main factor responsible for the a. Smoking
b. Alcohol use
decline in IMR during the 1990s?
c. Lack of physical activity
a. Improvements in medical care d. Age
b. Reduction in sudden infant death e. Obesity
syndrome
c. Reduction in vaccine-preventable
325. A mother brings in her one-
diseases
d. Advances in prenatal diagnosis
year-old child because she is con-
e. Reduction in the incidence of low cerned about potential exposure to
birth weight lead. They have been making reno-
vations in their older home and she
323. Disparities in IMR persist is now considering moving to
among socioeconomic groups. Com- another house until the work is
pared with white women, the IMR completed. You want to check the
for African American women is child s blood lead level. Which of
a. 25% higher
the following is the most accurate
b. 50% higher method of screening for lead poi-
c. 100% higher soning?
d. 50% lower a. Erythrocyte protoporphyrin
e. 75% lower b. Capillary blood lead
c. CBC
d. Venous blood lead
e. Ferritin
156 Preventive Medicine and Public Health

326. Which of the following types Items 328-330


of cancer is the most frequent cause
A50-year-old woman comes for
of gynecologic cancer deaths?
her periodical health examination.
a. Ovarian Her body mass index is 29 kg/m2.
b. Cervical
Her blood pressure is 120/80. She
c. Endometrial
has no family history of cardiovascu-
d. Vaginal
e. Vulvar lar disease. Her total cholesterol is
200 mg/dL (5.2 mmol/L), her HDL
327. You are employed by a gov- is 35 mg/dL (0.9 mmol/L), and her
ernment agency in the United States LDL is 100 mg/dL (2.6 mmol/L).
and are asked to make decisions
about allocating funds for disease 328. This patient is at highest risk
prevention. You consider the lead- for developing which of the follow-
ing causes of death in the United ing conditions?
States as guidance. In which order a. Stroke
would you prioritize allocation of b. Coronary artery disease
funds? c. Non-insulin-dependent diabetes
d. Pulmonary embolism
a. Heart disease, cancer, stroke, and e. Hypertension
chronic obstructive pulmonary dis-
ease (COPD)
329. This patients weight increases
b. Heart disease, cancer, COPD, and
stroke the risk for which of the following
c. Heart disease, COPD, cancer, and cancers?
stroke a. Breast, pancreas, and ovary
d. Cancer, heart disease, COPD, and b. Endometrium, breast, and colon
stroke c. Ovary, cervix, and colon
e. Accidents, COPD, heart disease, d. Cervix, ovary, and breast
and stroke e. Colon, endometrium, and ovary

330. The most appropriate initial


intervention for weight loss is
a. Exercise
b. Surgery
c. Appetite-suppressive drugs
d. Diet with less than 25% of total
calories from fat
e. Restriction to three meals per day
Noncommunicable and Chronic Disorders 157

331. The effectiveness of the nico- 333. Which of the following is the
tine patch for smoking cessation in- most important risk factor for devel-
creases with the intensity of the oping cervical cancer?
counseling provided. What percent- a. Coitarche before age 18
age of smokers who use the patch b. Herpes simplex virus infection
and receive intensive counseling is c. Multiple sexual partners
still abstinent 6 months after the end d. More than five years since the last
of the treatment? Pap smear
e. Human papillomavirus type 16
a. 10%
b. 25%
c. 40% 334. Above which level of desired
d. 55% body weight can someone be de-
e. 70% scribed as obese?
a. 110%
332. A mother brings her 14-year- b. 120%
old daughter to your office because c. 130%
she is concerned about her childs d. 140%
eating patterns. Her nutritional his- e. 150%
tory reveals that she generally eats
very little because she says she is not
hungry. She occasionally engages in
junk food binges with friends. She is
often constipated. She exercises reg-
ularly. She is 5′6″ tall and weighs
108 pounds. Her menarche was at
age 13. She stopped having periods
4 months ago. She says she has no
concerns about her body image and
thinks her mother is exaggerating
because everyone in the family is tall
and thin. The history and findings
are most likely associated with
a. Typical adolescent behavior
b. Depression
c. Hyperthyroidism
d. Bulimia
e. Anorexia
158 Preventive Medicine and Public Health

Items 335-336 337. Which of the following pa-


tients is at highest risk for develop-
In 1999, the CDC published
ing colon cancer?
the latest data for abortion surveil-
lance (1996) in the United States. a. A 50-year-old male with a long his-
tory of a diet high in animal fat
The abortion rate was 20/1000
b. A 45-year-old female with irritable
women aged 15 to 44 years, the
bowel syndrome
lowest since 1975. Mortality con- c. A 30-year-old with a history of
tinues to be very low, with a case- familial polyposis
fatality rate of less than 1/100,000 d. A 35-year-old male diagnosed with
legal abortions. Monitoring abor- ulcerative colitis at age 25
tion rates is useful for identifying e. A 45-year-old obese female with a
women at high risk of unintended diet low in fiber
pregnancy and evaluating effective-
ness of family-planning programs. 338. The most frequent cause of
death from unintentional injury in
335. Among which age group is children under the age of 12 months
the highest abortion ratio? is
a. <15 years a. Automobile accidents
b. 1519 years b. Falls
c. 2024 years c. Poisoning
d. 3539 years d. Asphyxiation
e. >40 years e. Fire

336. Approximately what percent- 339. The leading cause of death in


age of abortions are performed males aged 25 to 44 in the United
before 13 weeks of gestation? States is
a. 25% a. Heart disease
b. 50% b. Cancer
c. 60% c. HIV infection
d. 80% d. Homicide
e. 90% e. Accidents
Noncommunicable and Chronic Disorders 159

340. Which of the following 60- 343. You are asked to give a lec-
year-old patients is most likely to ture on the epidemiology of cancer
have an ischemic stroke within a in the United States at your public
year? health school alumni association.
a. A male smoker Which of the following statements
b. A male with hypertension best reflects the overall trends in
c. A male with an asymptomatic the United States?
carotid bruit a. Incidence of cancer, as well as mor-
d. A female with cardiovascular dis- tality rates, has been increasing in
ease children
e. A female with diabetes type 2 b. Incidence of lung cancer has been
increasing in adults, but mortality
341. A 50-year-old alcoholic male rates have decreased
presents to the emergency room c. Incidence of cancer has remained
with upper gastrointestinal (GI) stable in children, but mortality
bleeding. Examination reveals rates have decreased
ataxia, confusion, and ophthalmo- d. Incidence of breast cancer in women
plegia. In addition to treating the has increased as well as mortality
e. Incidence of prostate cancer in men
GI bleeding, he would benefit from
has increased as well as mortality
receiving which of the following?
a. Niacin 344. A 35-year-old woman pre-
b. Pyridoxine
sents to your office complaining of
c. Folic acid
d. Thiamine
hair loss, bone pain, and dryness
e. Cobalamin and fissures of the lips. She tells
you that she has been taking large
342. In the 1990s, what propor- amount of vitamins in hopes of pre-
tion of adults in the United States venting cancer and infections. Her
was reported to be overweight? symptoms are most likely caused
by an excess of
a. <5%
b. 1015% a. Vitamin A
c. 2025% b. Vitamin E
d. 2530% c. Vitamin C
e. 3035% d. Vitamin D
e. Vitamin K
160 Preventive Medicine and Public Health

Items 345-346 Items 347-349


A 40-year-old man presents for You have just accepted a posi-
his periodic health examination. He tion as medical director for a large
is overweight. His fasting blood group practice and plan to develop
sugar is 90 mg/dL, his total choles- guidelines for the provision of pre-
terol is 210 mg/dL with a high- ventive services. You plan to use
density lipoprotein (HDL) of 50 mg/ evidence-based medicine and follow
dL and a low-density lipoprotein the USPS Task Force recommenda-
(LDL) of 130 mg/dL. His blood tions. For each of the following inter-
pressure is 120/80. He does not ventions applied to the general
smoke. He has no cardiovascular or population you will serve, choose the
pulmonary symptoms. He admits most appropriate group to screen.
to being a couch potato and not
always eating healthily. You coun- 347. Total cholesterol measure-
sel him about increasing his physi- ments
cal activity and improving his diet. a. All men, women, and children
b. All men and women between the
345. For this patient, which of the ages of 45 and 64
following will most likely benefit c. Only men between the ages of 35
from physical activity? and 64
d. Men between the ages of 35 and 64
a. Blood pressure
and women between the ages of 45
b. Total cholesterol
and 64
c. HDL
e. All men and women, regardless of
d. LDL
age
e. Weight

348. Blood pressure measurements


346. Which of the following exer-
cise regimens would be most ap- a. All men, women, and children
propriate to begin with? b. All men and women
c. Men and women starting at age 20
a. Resistance training for 30 minutes d. Men and women starting at age 30
3 times a week e. Men and women starting at age 40
b. Jogging for 15 minutes 4 times a
week
c. Brisk walking for 30 minutes 3 times
a week
d. Jogging 60 minutes per day
e. Resistance training for 15 minutes
every day
Noncommunicable and Chronic Disorders 161

349. Mammography 352. Which of the following men-


a. Baseline at age 35, then yearly start- tal disorders is more likely to occur
ing at age 40 in men compared to women?
b. Yearly starting at age 40 a. Affective disorders
c. Yearly starting at age 50 b. Anxiety disorders
d. Every three years starting at age 40 c. Nonaffective psychosis
e. Every three years starting at age 50 d. Substance abuse or dependence
e. Simple phobia
Items 350-351
Breast cancer is the most fre- 353. A 75-year-old widowed
quent neoplasia among women. It woman is brought to the emergency
is estimated that 1 woman out of room because she fell while trying to
every 8 will develop the disease go to the bathroom. Her daughter
over her lifetime. states that she has been getting more
confused over the last few weeks.
350. What is the lifetime risk of She has been disabled by arthritis
developing breast cancer for a for many years. She lives with her
woman in whom the BRCA-1 has daughter who is single and works
been detected? full-time. The examination reveals
multiple ecchymoses on different
a. 20%
areas of the body. She is very under-
b. 40%
c. 60%
weight, but her daughter states that
d. 70% she refuses to eat. Which of the fol-
e. 80% or more lowing factors is most likely to cause
the clinical findings?
351. Which of the following fac- a. Cancer
tors is most likely to decrease the b. Abuse
lifetime risk of developing breast c. Alzheimers disease
cancer in women? d. Diabetes
e. Depression
a. Young age at menarche
b. Older age at menarche
c. Older age at menopause
d. Early menopause
e. Nulliparity
162 Preventive Medicine and Public Health

354. Which of the following find- 357. Which of the following fac-
ings is the the most consistent tors is associated with decreased
among offenders in cases of child drug use among young adults?
sexual abuse? a. Low socioeconomic status
a. Alcohol abuse b. Early drug use
b. Psychiatric illness c. Marriage
c. Stranger to the child d. Parental drug use
d. Prior sexual abuse e. Depression
e. Relative or known to the child
358. You are employed by a state
Items 355-356 substance abuse program and are
responsible for the design, imple-
A 27-year-old man is brought
mentation, and evaluation of drug
to the emergency room by his
prevention programs in schools.
friends because he has delusions
Which of the following attributes of
about being followed by the FBI
the programs is most likely to
and has paranoid thoughts and
impact drug use behavior?
behaviors.
a. Peer interaction
b. Length of program
355. Which of the following drugs
c. Expert instruction
is most likely to be causing this d. Size of the program
psychiatric presentation? e. Socioeconomic status of students
a. Cannabis
b. Heroin 359. A 20-year-old patient presents
c. LSD to the office for contraception coun-
d. Barbiturates
seling. Her history reveals no past
e. Cocaine
medical problems. Her physical and
pelvic examination is normal. She is
356. For what minimum length of
sexually active with the same part-
time should this patient be enrolled
ner for 9 months. Which of the
in a drug treatment program for
following contraceptive methods
positive outcomes to occur?
would be most appropriate?
a. 2 to 4 weeks
a. Barrier method
b. 1 to 3 months
b. Combined oral contraceptives
c. 4 to 6 months
c. Progestin-only contraceptives
d. 6 to 12 months
d. Intrauterine device (IUD)
e. 12 to 16 months
e. Barrier method and combined oral
contraceptives
Noncommunicable and Chronic Disorders 163

360. A 30-year-old patient presents 363. A 28-year-old patient GoPoAo


to your office for contraceptive comes for her first prenatal visit at 6
counseling. She is divorced and has weeks of pregnancy. Her examina-
just started a new relationship. She tion is normal for gestational age.
wishes to use oral contraceptives Her history reveals that she does not
because of their effectiveness. Her smoke. She drinks one glass of wine
examination is normal. Which of the about two days a week. She has
following factors would be consid- been married for three years and has
ered a contraindication to their use? only her husband as a sexual part-
a. Tobacco use ner. She is employed as a paralegal.
b. Vaginal bleeding Her family history is negative. She is
c. History of gestational hypertension very concerned about preserving the
d. Polycystic breast disease mental and physical health of her
e. Migraines baby. Which of the following inter-
ventions is most likely to reduce the
361. The use of oral contracep- risk of neurological defect in the
tives will have the most impact on infant?
gynecological mortality by reduc- a. Eliminating alcohol use
ing the risk for which of the follow- b. Folic acid supplements
ing cancers? c. Ultrasound
a. Ovarian d. Amniocentesis
b. Breast e. Alpha fetoprotein testing
c. Cervical
d. Vulvar 364. A 57-year-old woman pre-
e. Endometrial sents to the office because of vaginal
bleeding. She had her menopause at
362. Environmental tobacco smoke age 50. She does not use hormonal
(ETS) is a major public health issue replacement therapy. Her last peri-
in the United States, particularly for odic health examination was one
children. Which of the following year ago. The physical and pelvic
conditions is most affected by ETS? examinations are nomal. Which of
a. Low birth weight the following is the most likely diag-
b. Middle ear infection nosis?
c. Bronchitis or pneumonia a. Atrophic vaginitis
d. Asthma b. Blood coagulation disorder
e. Lung cancer c. Endometrial carcinoma
d. Cervical carcinoma
e. Ovarian cancer
164 Preventive Medicine and Public Health

365. Which of the following groups Items 367-369


of risk factors has been associated
A 53-year-old woman presents
with endometrial cancer?
to your office with questions about
a. Hypertension, diabetes, and obesity hormonal replacement therapy
b. Family history, obesity, and nulli-
(HRT). She has been experiencing
parity
hot flashes and night sweats. She has
c. Hypertension, oral contraceptives,
and nulliparity not menstruated for one year. She
d. Family history, early pregnancy, has no risk factors for cardiovascular
and diabetes disease. She is 5′6″ and weighs 120
e. Multiple pregnancies, obesity, and lbs. Her gynecological examination
family history is normal as well as her Pap smear.
Her breast examination and mam-
366. A 50-year-old woman pre- mography are also normal. She won-
sents to the office complaining of ders about the risks and benefits of
abdominal pain and bloating. The HRT given her health status.
pelvic examination reveals an ad-
nexal mass of 5 cm. An ultrasound 367. Which of the following lipid
confirms the findings of a solid alterations are associated with
right ovarian mass of 7 cm. Ovarian menopause?
cancer is suspected. Which of the a. Decrease in total and HDL choles-
following statements about the risk terol
and screening for this disease is b. Increase in total and LDL choles-
correct? terol
a. Routine screening for ovarian can- c. Increase in HDL cholesterol, but no
cer with CA-125 is recommended effect on total cholesterol
b. Screening with ultrasound is rec- d. Increase in LDL cholesterol, but no
ommended effect on total cholesterol
c. Family history is the most impor- e. No effect on HDL, LDL, or total
tant risk factor for developing the cholesterol
disease
d. Oral contraceptives increase the 368. HRT most increases her risk
risk of disease of developing which of the follow-
e. Most patients are diagnosed early ing conditions?
in the disease a. Hypertension
b. Thrombosis
c. Alzheimers disease
d. Gallbladder disease
e. Endometrial cancer
Noncommunicable and Chronic Disorders 165

369. The most benefit to be gained 372. Which of the following state-
by HRT for this patient will be ments best reflects our current
a. Reduction of osteoporosis knowledge about prostate cancer?
b. Reduction of cardiovascular disease a. African American men are at lower
c. Reduction of vasomotor symptoms risk of developing the neoplasia
d. Reduction in the risk of breast can- b. Prostate-specific antigen (PSA) is a
cer sensitive screening tool
e. Reduction in the risk of glucose in- c. PSA is a specific screening tool
tolerance d. Digital rectal examination (DRE)
can be helpful in detecting disease
370. Which of the following is the e. Metastasis and an aggressive course
leading cause of death for women is common
of all ages in the United States?
a. Ischemic heart disease Items 373-374
b. Lung cancer The National Cholesterol Edu-
c. Breast cancer cation Program has developed a
d. Accidents
two-step diet plan for persons with
e. Stroke
high cholesterol. Persons are con-
sidered at moderate risk of devel-
371. For which of the following
oping cardiovascular disease if their
patients would aspirin chemopro-
total cholesterol is between 200
phylaxis be CONTRAINDICATED?
and 239 mg/dL (5.26.2 mmol/L)
a. An asymptomatic 52-year-old man and at high risk if their total choles-
at risk of CAD
terol is 240 mg/dL or more (>6.2
b. An asymptomatic 60-year-old man
with a prior myocardial infarction
mmol/L).
c. A 55-year-old man with chronic
stable angina 373. Which of the following dietary
d. A 65-year-old man who has sur- restrictions could be associated with
vived unstable angina and a myocar- a decrease in HDL?
dial infarction a. Total dietary fat
e. A 45-year-old man with uncon- b. Total dietary cholesterol
trolled hypertension c. Total dietary carbohydrate
d. Total dietary polyunsaturated fat
e. Total dietary protein
166 Preventive Medicine and Public Health

374. After one of your patients fails 377. Which of the following is the
to reduce her total blood cholesterol most important risk factor for
on the step one diet, you counsel developing insulin-dependent dia-
her to start the step two diet. Which betes mellitus (IDDM)?
of the following modifications rep- a. Country of residence
resent the main difference between b. Male gender
the step one and the step two diet? c. Older age
a. Dietary intake of total calories d. Gestational diabetes in the mother
b. Dietary intake of total carbohy- e. Presence of HLA-DR3
drates
c. Dietary intake of saturated fat and 378. Which of the following com-
cholesterol plications is most likely to occur in
d. Dietary intake of total protein a 35-year-old patient diagnosed
e. Dietary intake of total fat with IDDM at age 15?
a. Retinopathy
375. Which of the following types b. Renal disease
of diets may reduce the risk of de- c. Neuropathy
veloping cancer? d. Stroke
a. Low-fiber diet e. Myocardial infarction
b. High-protein diet
c. High-fat diet 379. Which of the following dis-
d. Diet rich in vitamin C and eases is the leading cause of end-
β-carotene stage renal disease (ESRD)?
e. Diet rich in vitamin E
a. Hypertension
b. Pyelonephritis
376. Which of the following state- c. Diabetes
ments best reflects the epidemiol- d. Glomerulonephritis
ogy of hypertension in the United e. Obstructive nephropathy
States?
a. Systolic blood pressure tends to de-
crease with age
b. Alcohol and salt intake do not af-
fect blood pressure
c. Family history is an important risk
factor for developing hypertension
d. Obesity is not associated with hy-
pertension
e. Only control of diastolic blood pres-
sure has been shown to decrease
mortality
Noncommunicable and Chronic Disorders 167

380. Asthma is a common disease 382. A 25-year-old woman pre-


in children and the prevalence in the sents to the delivery room in labor.
United States was estimated at 5% in She has had no prenatal care. The
1992, a rise from 3% in 1982, with female newborn weighs 4.5 pounds
sharp declines noted from early and has episodes of seizures shortly
childhood to adolescence. Which after birth. Irritability and hyper-
of the following factors is most tonicity are also noted. The most
strongly predictive of mortality due likely cause for these findings in the
to asthma in children? newborn is
a. Age a. Cocaine use by mother
b. Gender b. Alcohol consumption by mother
c. Environmental pollutants c. HIV in the mother
d. Overdependence on nebulizers d. Syphilis in the mother
e. Severity of illness e. Heroin use by mother

381. A 52-year-old patient with 383. Postmenopausal women who


chronic cough and shortness of are not on hormone replacement
breath is diagnosed with chronic therapy can benefit from daily cal-
obstructive lung disease. Which of cium intake to reduce the risk of
the following factors is the most fractures secondary to osteoporo-
important contributor to this find- sis. Which of the following is the
ing? recommended amount of calcium
a. Tobacco use to be consumed daily?
b. Deficiency of α-antitrypsine a. 500 mg
c. Asthma b. 750 mg
d. Repeated childhood respiratory c. 1000 mg
tract infections d. 1500 mg
e. Occupation e. 2000 mg
168 Preventive Medicine and Public Health

384. Consider the following skin lesion.

(Reproduced, with permission, from Fauci AS, Braunwald E, Isselbacher


KJ, eds., Harrison s Principles of Internal Medicine, 14th ed., New York,
McGraw-Hill, 1998.)

Which of the following statements best describes the epidemiology in the


United States?
a. The incidence has been decreasing in recent years
b. Persons with fair complexions are at higher risk
c. Sunburn is not associated with the development of this lesion
d. Hereditary factors are not associated with this lesion
e. This lesion occurs primarily in children

385. A 22-year-old male presents to the student health center complaining


of scrotal discomfort and swelling. He has no complaint of urethral dis-
charge, fever, or genital lesions. He has been sexually active with the same
partner for 3 years and uses condoms regularly as their method of birth con-
trol. He is otherwise healthy. The examination reveals a tender mass in the
posterior aspect of the left testis. The most likely diagnosis is
a. Epididymitis
b. Lymphoma
c. Primary germ cell tumor
d. Varicoceles
e. Spermatoceles
Noncommunicable and Chronic Disorders 169

386. Which of the following pa- 389. Public health efforts to pre-
tients is at highest risk of adverse vent injuries have been particularly
effects from iron deficiency anemia? successful in reducing deaths from
a. A postmenopausal woman a. Firearms
b. An elderly widow living alone b. Fire
c. A breast-fed one-month-old infant c. Motor vehicle accidents
d. A 10-month-old with a diet of cows d. Falls
milk e. Hypothermia
e. A 14-year-old with heavy periods
390. Biological basis for the occur-
387. Which of the following public rence of mental illnesses has been
health interventions has been the the subject of a number of studies.
most successful in preventing initi- Schizophrenia has been linked with
ation of smoking or reducing the an increased activity of which of
prevalence of smoking? the following neurotransmitters?
a. Media campaigns against smoking a. Acetylcholine
b. Prohibiting the sale of tobacco to b. Dopamine
minors c. Serotonin
c. Restrictions on indoor smoking d. GABA (gamma aminobutyric acid)
d. Lawsuits against the tobacco indus- e. Norepinephrine
try
e. Increases in cigarette prices through 391. Which of the following acts
taxes as a cofactor in duodenal ulcer?
a. Cigarette smoking
388. Which of the following state-
b. Alcohol use
ments most accurately describes de- c. NSAID use
pressive disorders? d. Blood group O
a. They are associated with more fre- e. Helicobacter pylori
quent visits for physical symptoms
b. They mostly affect young married
men
c. They are rarely encountered in am-
bulatory care
d. They can result in suicide in over
50% of cases
e. They are not a major economic bur-
den in the United States
170 Preventive Medicine and Public Health

392. Consider the following pop- 394. A 68-year-old man with con-
ulation pyramid. trolled hypertension complains of
gradual impairment of vision. His
history further reveals that he was
Age
80+ Male Female recently diagnosed with mild adult
onset diabetes that is also well con-
trolled. He is a retired fisherman.
40Ð44 The most likely cause of his visual
impairment is
a. Glaucoma
b. Cataract
c. Diabetic retinopathy
9 6 3 3 6 9 d. Macular degeneration
Percent
e. Xerophthalmia

This pyramid is representative of 395. A man afflicted with neuro-


which population structure? fibromatosis is the parent of a
a. Stationary healthy, unafflicted female child.
b. Expansive What is the probability that this
c. Constrictive child will transmit the disease to
d. Low fertility her own offspring if her partner is
e. High mortality asymptomatic of the disease?
a. 25%
393. A 22-year-old female pre- b. 50%
sents with a history of abdominal c. 100%
cramps, diarrhea, weight loss, and d. 0%
blood and mucus in the stools. e. 75%
Which of the following is a risk fac-
tor for developing this condition? 396. Which of the following ge-
a. Oral contraceptives netic abnormalities is responsible
b. Stress for most cases of untreatable severe
c. Smoking mental retardation?
d. High-fat diet a. Sex chromosome disorders
e. Coffee b. Autosomal chromosome disorders
c. X-linked recessive disorders
d. Autosomal dominant disorders
e. X-linked dominant disorders
Noncommunicable and Chronic Disorders 171

1 is the
397. Which of the following Items 402-404
most cost-effective and safe public
Match the following behavior
health measure today to prevent
modification descriptions with the
dental caries?
appropriate theoretical models.
a. Water fluoridation
a. Health belief model
b. Proper nutrition
b. Social learning theory
c. Regular dental visits
c. Theory of planned behavior
d. Promotion of regular flossing
d. Theory of triadic influence
e. Promotion of regular brushing
e. Stages of change theory

Items 398401
402. Precomtemplative, comtem-
Match each set of symptoms plative, and ready for action.
and signs with the dietary defi- Stages of change
ciency. 403. Perceived susceptibility, sever-
a. Vitamin A deficiency ity, and benefits. health belief model
b. Thiamine deficiency
c. Vitamin C deficiency 404. Outcome and efficacy expec-
d. Vitamin D deficiency tations. social learning theory
e. Niacin deficiency
f. Vitamin E deficiency Items 405-408
g. Vitamin K deficiency
For each description below,
398. Petechiae, sore gums, hema- match the most appropriate level of
C turia, and bone or joint pain prevention.
a. Primary prevention
399. Dermatitis, diarrhea, and de- b. Secondary prevention
lirium. Niacin c. Tertiary prevention
d. Primary and tertiary prevention
400. Edema, neuropathy, and myo- e. Primary and secondary prevention
f. Secondary and tertiary prevention
cardial failure.
Thiamine
405. Treating a pregnant woman
401. Conjunctival xerosis, hyper-
A keratosis, and keratomalacia.
infected with syphilis. 1, 3

406. Using condoms during sex-


ual intercourse. 1
172 Preventive Medicine and Public Health

407. Pasteurizing milk. 1 413. Tobacco use.

408. Screening for hypertension. 414. Family history.


1, 2
Items 409-412 415. Alcohol.
Match the effect of deficiency
Items 416-418
to the proper mineral.
a. Fluorine Match the ages with their lead-
b. Copper ing causes of death.
c. Zinc a. Cancer
d. Sodium b. Heart disease
e. Calcium c. Suicide
d. Homicide
409. Poor mineralization of bones e. Injury
and teeth, osteoporosis. calcium f. HIV/AIDS
g. Stroke
410. Nausea, diarrhea, muscle
cramps, dehydration. sodium 416. Ages 11 to 24. Injury

411. Tendency to dental caries. 417. Ages 25 to 64. cancer


fluorine
412. Dwarfism, hepatosplenomeg- 418. Age 65 and older heart dse
aly, poor wound healing. zinc

Items 413-415
Match the risk factor well doc- Tobacco: cervical, lung,
umented for the type of cancer. esophageal
Choose one or more than one.
a. Endometrial cancer Family history: breast,
b. Cervical cancer
c. Breast cancer ovarian, colon
d. Ovarian cancer
e. Colon cancer Alcohol: breast,
f. Lung cancer
g. Prostate cancer
esophageal
h. Esophageal cancer
Noncommunicable and Chronic Disorders 173

Items 419-422 Items 423-424


Match the following popula- Match the typical use failure rate
tions to the disease which has a with the method of contraception.
higher prevalence. a. Combined oral contraceptives
a. Alpha-thalassemia b. Intrauterine device (IUD)
b. Beta-thalassemia c. Cervical cap
c. Cystic fibrosis d. Diaphragm
d. Hemophilia e. Condoms
e. Tay-Sachs disease f. Spermicide
f. Sickle cell disease g. Withdrawal
g. Hemoglobin E
423. 3%. Combined OCP
419. Southern Europe.
beta-thalassemia 424. <1%. IUD
420. Africa.
Sickle Cell
421. Asia. Hgb E

422. Ashkenazic Jews. Tay-Sachs

You might also like